Sei sulla pagina 1di 81

lOMoARcPSD|3982611

MATH3510-Actuarial Mathematics 1-Lecture Notes release

Actuarial Mathematics 1 (University of Leeds)

StuDocu is not sponsored or endorsed by any college or university


Downloaded by Kipkoech Stanley (stanleykipkoech8@gmail.com)
lOMoARcPSD|3982611

MATH3510: Actuarial Mathematics 1


Outline Lecture Notes

Georgios Aivaliotis and Jonty Carruthers (Student)


School of Mathematics
University of Leeds

December 14, 2016

Downloaded by Kipkoech Stanley (stanleykipkoech8@gmail.com)


lOMoARcPSD|3982611

Contents

1 Survival Models 2
1.1 Survival, distribution, and density functions . . . . . . . . . . . . . . . . . . . . . . . . 2
1.2 The force of mortality . . . . . . . . . . . . . . . . . . . . . . . . . . . . . . . . . . . . 4
1.3 Parametric classes and the future lifetime random variable . . . . . . . . . . . . . . . . 6
1.4 Deferred mortality probabilities . . . . . . . . . . . . . . . . . . . . . . . . . . . . . . . 8
1.5 Curtate and expected future lifetime . . . . . . . . . . . . . . . . . . . . . . . . . . . . 9
1.6 Temporary expected future lifetime . . . . . . . . . . . . . . . . . . . . . . . . . . . . . 11

2 Life Tables and Selection 14


2.1 Introduction to life tables . . . . . . . . . . . . . . . . . . . . . . . . . . . . . . . . . . 14
2.2 Uniform distribution of deaths . . . . . . . . . . . . . . . . . . . . . . . . . . . . . . . . 17
2.3 Constant force of mortality . . . . . . . . . . . . . . . . . . . . . . . . . . . . . . . . . 19
2.4 Select and ultimate life tables . . . . . . . . . . . . . . . . . . . . . . . . . . . . . . . . 20

3 Life Insurance Benefits 23


3.1 Introduction to life insurance functions . . . . . . . . . . . . . . . . . . . . . . . . . . . 23
3.2 Term and Whole life insurance . . . . . . . . . . . . . . . . . . . . . . . . . . . . . . . 24
3.3 Endowment and deferred insurances . . . . . . . . . . . . . . . . . . . . . . . . . . . . 27
3.4 Insurance functions for De Moivre’s model . . . . . . . . . . . . . . . . . . . . . . . . . 28
3.5 Examples - continuous insurances . . . . . . . . . . . . . . . . . . . . . . . . . . . . . . 30
3.6 Discrete time insurance functions . . . . . . . . . . . . . . . . . . . . . . . . . . . . . . 32
3.7 Recursive Relationships . . . . . . . . . . . . . . . . . . . . . . . . . . . . . . . . . . . 35
3.8 Benefit payments for mthly periods . . . . . . . . . . . . . . . . . . . . . . . . . . . . . 38
3.9 Continuous and discrete relationships . . . . . . . . . . . . . . . . . . . . . . . . . . . . 40
3.10 Variable insurance benefits . . . . . . . . . . . . . . . . . . . . . . . . . . . . . . . . . 42

4 Life Annuities 45
4.1 Introduction to life annuities . . . . . . . . . . . . . . . . . . . . . . . . . . . . . . . . . 45
4.2 Temporary and deferred life annuities . . . . . . . . . . . . . . . . . . . . . . . . . . . 47
4.3 Guaranteed annuities . . . . . . . . . . . . . . . . . . . . . . . . . . . . . . . . . . . . . 50
4.4 The UDD assumption for life annuities . . . . . . . . . . . . . . . . . . . . . . . . . . . 53
4.5 Life annuities payable continuously . . . . . . . . . . . . . . . . . . . . . . . . . . . . . 55
4.6 Increasing Annuities . . . . . . . . . . . . . . . . . . . . . . . . . . . . . . . . . . . 58

5 Premium Calculation and Policy Values 61


5.1 The Equivalence Principle . . . . . . . . . . . . . . . . . . . . . . . . . . . . . . . . . . 61
5.2 The Equivalence principle (continued) . . . . . . . . . . . . . . . . . . . . . . . . . . . 65
5.3 Policy Values and Reserves . . . . . . . . . . . . . . . . . . . . . . . . . . . . . . . . . 71

Illustrative Life Table

Downloaded by Kipkoech Stanley 1


(stanleykipkoech8@gmail.com)
lOMoARcPSD|3982611

Chapter 1

Survival Models

1.1 Survival, distribution, and density functions


When considering a life insurance policy the most important factor is determining how long the
inidividual is likely to survive. In this chapter, survival models, including those by Makeham and
De Moivre, are introduced to provide a method of computing survival and mortality probabilities for
varying ages. An introduction to the notation used in actuarial science is also provided, parts of which
may have already been seen in MATH2775 Survival Analysis. The future lifetime and curtate future
lifetime random variables are defined and it is these that will form a vital part of the insurance and
annuity functions defined in later chapters.

Let X be a continuous and non-negative random variable, then X is an appropriate random variable
to model the age at which an individual dies and is therefore called the age at death random variable.

Definition: The survival distribution function of X is the probability that a newborn survives to at
least age x and is defined as
S0 (x) = Pr(X > x) (1.1)

This function is also known as the decumulative distribution function but more commonly it is simply
referred to as the survival function. For a function to be a legitimate survival function it must satisfy
all three of the following properties

• S0 (0) = 1,

• limx→∞ S0 (x) = 0,

• S0 (x) is a non-increasing function of x.

Example 1: Consider the proposed survival function


 1
x 2
S0 (x) = 1 − 100 0 ≤ x ≤ 100

Determine whether this function is a legitimate survival function.

Solution: We need to check that S0 (x) satisfies the three conditions previously given. By substituting
for x = 0 it can be seen that S0 (0) = 1, and similarly for x = 100, S0 (100) = 0. To determine whether
the function is non-increasing we consider the first derivative. Differentiating once with respect to x
gives
−1
 1
x −2
S0′ (x) = 200

1 − 100

Downloaded by Kipkoech Stanley 2


(stanleykipkoech8@gmail.com)
lOMoARcPSD|3982611

CHAPTER 1. SURVIVAL MODELS 3

Since S0′ (x) is non-positive for all x in the given domain, S0 (x) must be non-increasing. All three
properties are satisfied so we conclude that S0 (x) is a legitimate survival function.

Note that in the previous example there is a slight alteration to the second property and the limit is
taken as x tends to 100 as opposed to infinity, this is becasue the survival function is only defined
for x ≤ 100. In survival models a limiting age is often defined since it is not necessary to consider
unrealistically large values of x. The limiting age is denoted by ω and is usually between 100 and 120,
the limiting age in the previous example is ω = 100.

Definition: The cumulative distribution function of X is the probability that a newborn dies before
age x and is given by
F0 (x) = Pr(X ≤ x) (1.2)
This function is more often referred to as the distribution function and as with the survival function
there are three properties that any legitimate distribution function must satisfy. These properties are
• F0 (0) = 0,
• limx→∞ F0 (x) = 1,
• F0 (x) is a non-decreasing function.
Since surviving beyond age x and dying before age x are complementary events, there exists a simple
relationship between the survival and distribution functions, namely,

F0 (x) = 1 − S0 (x) (1.3)

The final function used to calculate mortality probabilities is the probability density function (p.d.f).

Definition: Assume that for the r.v. X there exists a continuously differentiable survival function
S0 (x), then X has a continuous distribution function F0 (x) and the probability density function may
be defined as
d
f0 (x) = F (x)
dx 0

a standard result that should be recognised from previous courses in probability.

Given the p.d.f there also exist integral expressions for the survival and distribution functions

R∞ Rx
S0 (x) = x
f0 (s) ds F0 (x) = 0
f0 (s) ds

Occasionally a question may ask to find the p.d.f given only the survival function, in which case an
obvious approach may be to first calculate the distribution function and then differentiate to find the
p.d.f. However, by using equation (1.3) we can obtain the following expression for the density in terms
of the survival function.

d d d
f0 (x) = F0 (x) = (1 − S0 (x)) = − S0 (x) (1.4)
dx dx dx

The following example demonstrates how the functions defined above can be used to calculate mortality
probabilities.

Downloaded by Kipkoech Stanley (stanleykipkoech8@gmail.com)


lOMoARcPSD|3982611

CHAPTER 1. SURVIVAL MODELS 4

Example: Consider the survival function


1
1
S0 (x) = 10
[100 − x] 2 for 0 ≤ x ≤ 100

What is the probability that a newborn dies between the ages of 65 and 75?

Solution: The required probability is

Pr(65 ≤ X ≤ 75) = Pr(X ≤ 75) − Pr(X ≤ 65) = F0 (75) − F0 (65)


= (1 − S0 (75)) − (1 − S0 (65)) = S0 (65) − S0 (75)
= 0.5 − 0.40839 = 0.0916.


1.2 The force of mortality


The force of mortality is an important concept in actuarial science and the modelling of future lifetimes,
it considers the probability of death within an infinitesimal interval that we shall denote by ∆t. The
force of mortality may also be referred to as the force of failure or the hazard rate.

Definition: For a life aged x the force of mortality is defined as


1
µx = lim Pr (X ≤ x + ∆x|X > x) (1.5)
∆x→0 ∆x

Let µ(x) be a non-negative real-valued function, µ(x) may be considered as a force of mortality if and
only if the following properties are satisfied

• for all x ≥ 0, µ(x) ≥ 0


R∞
• 0 µ(x) dx = ∞

From the definition of the force of mortality, we can obtain an equation linking it with both the survival
function and p.d.f. The following derivation uses the formal definition of a derivative seen in second
year analysis courses in addition to equation (1.4) from Section 1.
1
µx = lim Pr (X ≤ x + ∆x|X > x)
∆x→0 ∆x
1 Pr(X > x) − Pr(X > x + ∆x)
= lim
∆x→0 ∆x Pr(X > x)
1 S0 (x) − S0 (x + ∆x)
= lim
∆x→0 ∆x S0 (x)
1 S0 (x + ∆x) − S0 (x)
=− lim
S0 (x) ∆x→0 ∆x
1 d
=− S0 (x)
S0 (x) dx

f0 (x)
=⇒ µx = (1.6)
S0 (x)
By an application of the chain rule, an alternative equation for the force of mortality is
1 d d
µx = − S0 (x) = − log (S0 (x)) (1.7)
S0 (x) dx dx

Downloaded by Kipkoech Stanley (stanleykipkoech8@gmail.com)


lOMoARcPSD|3982611

CHAPTER 1. SURVIVAL MODELS 5

Example: The distribution function for a survival model is


 x  61
F0 (x) = 1 − 1 −
120
Find an expression for the force of mortality µx .

Solution: The respective survival and density functions are


 x  16 1  x − 65
S0 (x) = 1 − and f0 (x) = 1−
120 720 120
Therefore, the force of mortality may written as
1  x −1 1 1
µx = 1− = =
720 120 6(120 − x) 720 − 6x


Note that it is easier to solve this example using equation (1.6) instead of (1.7) so it is important to learn
both. Equation (1.7) is only particularly useful when the survival function is written as an exponent,
however, it can also be used to derive one of the most important formulae in the course. Integrating
equation (1.7) between 0 and y gives
Z y
µx dx = − [log (S0 (y)) − log (S0 (0))]
0

but since S0 (0) = 1, it follows that log (S0 (0)) = 0 and therefore
Z y  Z y 
µx dx = −log (S0 (y)) ⇐⇒ S0 (y) = exp − µx dx
0 0

In keeping with the notation previously used, a simple exchange of variables gives
 Z x 
S0 (x) = exp − µs ds (1.8)
0

We now give an example demonstrating an application of this equation.

Example: A survival model is defined with force of mortality µs = ksn for constants k and n such that
s ≥ n, k > 0 and n ≥ 1. Find an expression for the survival function S0 (x).

Solution: Substituting the expression for the force of mortality into equation (1.8) gives
 Z x 
n
S0 (x) = exp − ks ds
0
 
k  n+1 x
= exp − s 0
n+1
n+1
 
kx
= exp −
n+1


The force of mortality in the previous example is in fact the force of mortality for the Weibull model,
one of a number of parametric models defined in the following section.

Downloaded by Kipkoech Stanley (stanleykipkoech8@gmail.com)


lOMoARcPSD|3982611

CHAPTER 1. SURVIVAL MODELS 6

1.3 Parametric classes and the future lifetime random variable


De Moivre’s Law
De Moivre’s law is a simple law of mortality based on a linear survival function where the age at death
random variable X follows a uniform distribution between 0 and the limiting age ω, that is X ∼ U(0, ω).
Recall that for Z ∼ U(a, b), the p.d.f is
1
f (z) =
b−a
Under De Moivre’s model the p.d.f is therefore

1
f0 (x) =
ω

Using this, along with relations defined in previous sections, expressions can be obtained for the survival
function, distribution function, and force of mortality.
Z x Z x
1 x
F0 (x) = f0 (s) ds = ds =⇒ F0 (x) =
0 0 ω ω
ω ω
1
Z Z h z iz=ω x
S0 (x) = f0 (z) dz = dz = =⇒ S0 (x) = 1 −
x x ω ω z=x ω

f0 (x) 1 ω 1
µx = = . =⇒ µx =
S0 (x) ω ω−x ω−x

1
Example: Given that µx = 100−x
,0 ≤ x ≤ 100, find an expression for the survival function S0 (x).

Solution: There are two ways to approach this question, either we can recognise that the given force
of mortality is in the required form for De Moivre’s model with a limiting age ω = 100 and hence
x
S0 (x) = 1 − 100 , or, a safer approach is to find the expression explicitly using equation (1.8).

 Z x

1
S0 (x) = exp − dz = exp ([log(100 − z)]z=x
z=0 )
0 100 − z
= exp (log(100 − x) − log(100))
  x 
= exp log 1 −
100
x
=1−
100


The table on the following page provides a list of the key parametric models used in actuarial science
along with the corresponding survival function, force of mortality, and any limitations on the parameters
involved. Makeham’s law is another commonly used model since the force of mortality is comprised
of two components, a constant force that is independent of current age, and a force that increases
exponentially with age. Note that a special case of Makeham’s law is when A=0 and the model reduces
to Gompertz’s law.

Downloaded by Kipkoech Stanley (stanleykipkoech8@gmail.com)


lOMoARcPSD|3982611

CHAPTER 1. SURVIVAL MODELS 7

Law/distribution µx S0 (x) limitations


1 x
De Moivre (Uniform) ω−x
1− ω
0≤x≤ω

Exponential µ exp(−µx) µ>0


 
B
Gompertz Bcx x
exp − logc (c − 1) B > 0, c > 1
 
B
Makeham A + Bcx exp −Ax − logc (cx − 1) A ≥ −B, c > 1, B ≥ 0
 n+1 
Weibull kxn exp −kxn+1
x ≥ n ≥ 1, k > 0

The future lifetime random variable


So far we have only considered modeling future lifetime by considering the age at which the individual
dies, however, given an individual aged x, it is sometimes more appropriate to consider how much
longer they will survive for. We now introduce the notion of the future lifetime random variable, revise
our expressions for the survival and distribution functions and include the notation commonly used
throughout actuarial science.

Definition: If X is the age at death random variable and x is the current age of an individual, the
future lifetime can be defined by the random variable

Tx = X − x (1.9)

Consider an individual currently aged x who survives for another t years, the probability of this is given
by the survival function
Sx (t) = Pr(Tx > t)
Similarly, the probability that the individual dies within t years is given by the distribution function

Fx (t) = Pr(Tx ≤ t)

When we consider Sx (t) we make the underlying assumption that the life has survived from birth
through to age x before surviving from age x to x + t. This assumption allows us to write Sx (t) in
terms of the age at death random variable X and results in a very important equation relating S0 (x)
and Sx (t).
Pr(X > x + t)
Sx (t) = Pr(Tx > t) = Pr(X > x + t|X > x) =
Pr(X > x)
S0 (x + t)
=⇒ Sx (t) = (1.10)
S0 (x)

The actuarial notation used to define the survival and distribution functions is as follows, this notation
will be used in place of the existing notation throughout the remainder of the course.

Sx (t) = t px Fx (t) = t qx

In actuarial notation, the equation linking the survival and distribution functions may therefore be
written as
t p x + t qx = 1

Downloaded by Kipkoech Stanley (stanleykipkoech8@gmail.com)


lOMoARcPSD|3982611

CHAPTER 1. SURVIVAL MODELS 8

It is also common practice that in the case where t = 1 the first subscript is omitted and we simply
write px and qx for the survival and distribution functions.

The p.d.f of the future lifetime random variable is given in terms of the distribution and survival
functions in a similar way as before.
d d
fx (t) = Fx (t) = − Sx (t) (1.11)
dt dt
The force of mortality may also be defined for the future lifetime random variable as
1
µx+t = lim Pr(t < Tx < t + ∆t|Tx > t)
∆t→0 ∆t
Recall that equations (1.6) and (1.7) give two different expressions for the force of mortality of a life
aged x. Equivalent expressions also exist for the force of mortality at any age x+t, t > 0, the derivations
of which are similar to before but instead begin with the definition of µx+t as opposed to µx . For this
reason the derivations are not included but the results are as follows

fx (t) d
µx+t = and µx+t = − log (Sx (t))
Sx (t) dt
Finally we state without proof the analogue to equation (1.8) for calculating the survival function Sx (t)
given only the force of mortality. The proof may be attempted as an exercise and is a simple application
of equation (1.10) and (1.8).
 Z t 
Sx (t) = exp − µx+s ds (1.12)
0

1.4 Deferred mortality probabilities


Sometimes we may wish to consider the probability that an individual currently aged x survives for
some period of length t but dies in the subsequent u years, that is, the individual dies between the ages
x + t and x + t + u. This is called a deferred mortality probability and is a key part in the formulation of
the insurance functions that will be introduced in chapter 3. In actuarial notation the deferred mortality
probability is given by

t|u qx = Pr(t < Tx ≤ t + u) = Pr(Tx > t) − Pr(Tx > t + u) = t px − t+u px (1.13)

There also exists a second equation for the deferred mortality probability, but before we show this we
need to consider the multiplicative property of the survival function. Suppose we are interested in
the probability that a life currently aged x survives for another 2 years but only know the probability
of survival for each year separately. The probability of surviving for 2 years may be written as the
probability of surviving the first year, multiplied by the probability of surviving the second year. In
actuarial notation this may be written as

2 px = px · px+1

It is important to note that in general, the multiplicative rule only holds for the survival function and
not the distribution function, that is
t+u qx 6= t qx · u qx+t

The following example demonstrates how useful the multiplicative rule can be.

Example: You are given 5 p50 = 0.9, 10 p50 = 0.8, and q55 = 0.03. Find the probability that an individual

Downloaded by Kipkoech Stanley (stanleykipkoech8@gmail.com)


lOMoARcPSD|3982611

CHAPTER 1. SURVIVAL MODELS 9

currently aged 56 dies within the next 4 years.

Solution: We require the probability 4 q56 and we know that

10 p50 = 5 p50 · 5 p55


= 5 p50 · p55 · 4 p56
= 5 p50 · (1 − q55 )(1 − 4 q56 )

Substituting for the appropriate values and rearranging gives


0.8
4 q56 =1− = 0.08362
0.9(1 − 0.03)

Consider again the deferred probability of an individual aged x dying between x + t and x + u + t. The
individual survives the deferred period with probability t px and dies in the period of mortality with
probability u qx+t , therefore using the multiplicative rule, the deferred probability may be written as

t|u qx = t px · u qx+t (1.14)


It is important to know (1.13) and (1.14) since both are used in derivations later in the course and some
questions may only provide enough information to use one or the other.

1.5 Curtate and expected future lifetime


Curtate future lifetime random variable
A standard insurance contract comprises of two components, the premiums paid by the insured and the
benefits paid by the insurance company. Consider the case where an individual enters into a contract
and pays premiums at the beginning of every year throughout the duration of the contract. In this
case we are not interested in the exact time of death but whether or not the individual is alive at the
beginning of each year. The curtate future lifetime r.v is defined as the integer part of the future lifetime
r.v and can be thought of as the number of whole years the individual has survived for. We define the
curtate future lifetime r.v. using the floor function that takes the integer part of a real number and
then provide two properties with a short proof for each

Definition: For a life aged x, if Tx is the future lifetime random variable, the curtate future lifetime
random variable is defined as
Kx = ⌊Tx ⌋

The probability mass function (p.m.f) for Kx can be expressed as

Pr(Kx = k) = k |qx (1.15)

Proof:
Pr(Kx = k) = Pr(k ≤ Tx < k + 1)
= Pr(Tx > k) − Pr(Tx > k + 1)
= k px − k+1 px
= k |qx

Downloaded by Kipkoech Stanley (stanleykipkoech8@gmail.com)


lOMoARcPSD|3982611

CHAPTER 1. SURVIVAL MODELS 10


The distribution function for Kx is given by

Pr(Kx ≤ k) = k+1 qx (1.16)

Proof:
Pr(Kx ≤ K) = Pr(Kx = 0) + Pr(Kx = 1) + ... + Pr(Kx = k)
= 0| qx + 1| qx + ... + k| qx
k
X k
X
= j| qx = (j px − j+1 px ) using equation (1.13)
j=0 j=0
k
X k
X
= j px − j+1 px
j=0 j=0

= (0 px + px + ... + k px ) − (px + 2 px + ... + k+1 px )


= 1 − k+1 px (since 0 px = 1)
= k+1 qx


We do not derive an expression for the survival function of Kx as this can easily be deduced from
knowing the distribution function.

Expected future lifetime


We first consider the expected value of Tx , known as the complete expectation of life or expected future
lifetime, and denoted in actuarial notation by e̊x . To formulate an expression for e̊x we use the method
of integration by parts and recall the following formula for the expected value of a continuous random
variable X. Z ∞
E[X] = xf (x) dx, where f (x) is the density of X
−∞

The expected future lifetime is


Z ∞
e̊x = E[Tx ] = tfx (t) dt
0
Z ∞  
d
= t − t px dt using equation (1.11)
0 dt
Z ∞

= [t · t px ]0 − (−t px ) dt.
0

Since t px → 0 as t → ∞, evaluating the first term at each end point yields zero and so the only non-zero
term is the second term. Therefore,
Z ∞
e̊x = t px dt (1.17)
0

The second moment can be evaluated by considering


Z ∞
2
E[Tx ] = t2 fx (t) dt
0

which may be again evaluated using integration by parts to give


Z ∞
2
E[Tx ] = 2t · t px dt
0

Downloaded by Kipkoech Stanley (stanleykipkoech8@gmail.com)


lOMoARcPSD|3982611

CHAPTER 1. SURVIVAL MODELS 11

If both the first and second moments are known, the variance of Tx may be calculated using

Var[Tx ] = E[Tx2 ] − (E[Tx ])2 = E[Tx2 ] − (e̊x )2 . (1.18)

We may also be interested in the expected value of Kx , known as the curtate expectation of life and
denoted in actuarial notation by ex . To obtain an expression for ex we use the definition of the expected
value of a discrete random variable as follows

X ∞
X
E[Kx ] = kPr(Kx = k) = k · k| qx
k=0 k=0
X∞
= k (k px − k+1 px )
k=0

X ∞
X
= k · k px − k · k+1 px
k=0 k=0
= (px + 22 px + 33 px + ...) − (2 px + 23 px + 34 px + ...)
= px + 2 px + 3 px + ...

therefore,

X
ex = k px (1.19)
k=1

There also exists a relationship linking the curtate and standard expectations of life based on methods
from numerical analysis for evaluating definite integrals. Recall that the trapezium rule is based on the
approximation Z b  
f (a) + f (b)
f (x) dx ≈ (b − a)
a 2

Then from the definition of the expected future lifetime,


∞ ∞ Z k+1 ∞
1
Z X X
e̊x = t px dt = t px dt ≈ (k px + k+1 px )
0 k=0 k k=0
2
1
= [(0 px + px ) + (px + 2 px ) + (2 px + 3 px ) + ...]
2

!
1 X
= 1+2 k px
2 k=1

1 X
= + k px
2 k=1

Therefore, the approximation may be written as

1
e̊x ≈ + ex (1.20)
2

1.6 Temporary expected future lifetime


In the previous section we introduced the idea of expected future lifetimes, we now extend this further
and consider the temporary or partial expectation of life. The temporary expectation of life is the
number of years an individual is expected to survive between the ages of x and x + n, n > 0.

Downloaded by Kipkoech Stanley (stanleykipkoech8@gmail.com)


lOMoARcPSD|3982611

CHAPTER 1. SURVIVAL MODELS 12

The temporary expectation of life is denoted by e̊x:n and the curtate temporary expectation of life by
ex:n . The formulae remain the same as in the previous section with the exception of the upper limit
which changes from infinity to n, the formulae are therefore
Z n n
X
e̊x:n = t px dt and ex:n = k px
0 k=1

The following equation relates the temporary and standard expectations of life.

e̊x = e̊x:n + n px e̊x+n

Proof:
Z n Z ∞
e̊x:n + n px e̊x+n = t px dt + n px t px+n dt
0 0
Z n Z ∞
= t px dt + t+n px dt
0 0
Z n Z ∞
= s px ds + s px ds
0 n
Z ∞
= s px ds
0
= e̊x 

We finish this chapter with a worked example that makes use of some of the key points and equations
provided so far.

1
Example: A survival model has force of mortality µx = 2(100−x) , 0 ≤ x ≤ 100
(i) Find an expression for the survival function S0 (x).
(ii) Find an expression for the survival function t px .
(iii) Compute e̊36 , the expected future lifetime for an individual aged 36.

Solution: To obtain the survival function for part (i) we need to use equation (1.8) with the force of
mortality given in the question.
 Z x 
1
S0 (x) = exp − dz
0 2(100 − z)

  z=x 
1 h 1 z=x
i 
= exp − − log(100 − z) = exp log(100 − z) 2
2 z=0 z=0

  12 !
100 − x
= exp log
100

 x  12
= 1−
100

For part (ii) we can choose from one of two methods, we can either use equation (1.10) with our result
from part (i), or we can take a more direct approach using equation (1.12).

Downloaded by Kipkoech Stanley (stanleykipkoech8@gmail.com)


lOMoARcPSD|3982611

CHAPTER 1. SURVIVAL MODELS 13

For the first method we have


 21 1
1 − x+t

S0 (x + t) 100 100 − x − t 2
t px = = 1 =
S0 (x) x 2
1 − 100 100 − x

  21
t
= 1−
100 − x

Using the second method


 Z t 
1
t px = exp − ds
0 2(100 − x − s)

 s=t !
1
= exp − log(100 − x − s)
2 s=0

  12 !
100 − x − t
= exp log
100 − x

  12
t
= 1−
100 − x

the same result as found using the first method. Note that in this case the first method is only quicker
because we had already calculated S0 (x) earlier in the question. Finally, for part (iii) we simply need
to substitute our expression for t px into equation (1.17) with x = 36.
Z 64   21
t
e̊36 = 1− dt
0 64

Z 0 Z 1
1 1
= −64 u du = 64 2 u 2 du
1 0

128 h 3 iu=1
= u2
3 u=0

128
= ≈ 42.67
3
where the substitution u = 1 − 64t has been used to evaluate the integral. It is important to note that


in the initial integral the upper limit is 64 instead of infinity as stated in the general formula for e̊x .
This is because the survival model is only defined for x ≤ 100 and therefore for an individual currently
aged 36, they can only survive for a maximum of 64 years.

Downloaded by Kipkoech Stanley (stanleykipkoech8@gmail.com)


lOMoARcPSD|3982611

Chapter 2

Life Tables and Selection

2.1 Introduction to life tables


In this chapter we introduce the idea of a life table and how they may be used to evaluate survival
probabilities. Since we have so far only considered probabilities involving integer ages, we also discuss
two commonly used fractional age assumptions that allow us to compute probabilities for all ages and
durations. We finish the chapter by explaining the idea of selection and the effects this has on an
individual’s survival probabilities.

A life table is a tabular representation of mortality evolution for a group of lives. Initially the number
of lives is denoted by l0 , the radix of the life table, and is usually in excess of 1000. Let lx be the number
of individuals alive at age x, the probability that a newborn survives for x years is then the number of
individuals alive at age x as a fraction of the total number of lives at the start, that is,
lx
S0 (x) = x p0 = (2.1)
l0
The life table may not always begin with newborn lives and instead starts at some integer age x, if this
is the case we need an expression for the survival function Sx (t). We know that lx is the number of
individuals alive at age x, so lx+t is the number of individuals alive at age x + t. By the same reasoning
as before
lx+t
Sx (t) = t px = (2.2)
lx
The number of deaths that occur between x and x + 1 is calculated using

dx = lx − lx+1 (2.3)

and it is usual in integer tabulated life tables that this information is included in one of the columns.
The number of deaths between x and x + t is given by

t dx = lx − lx+t (2.4)

The probability that a life aged x dies within the next t years may be written as
lx+t lx − lx+t t dx
t qx = 1 − t px = 1 − = = (2.5)
lx lx lx
Some intuitive formulae that can be useful include

X t−1
X
lx = dx+k and t dx = dx+k
k=0 k=0

Downloaded by Kipkoech Stanley14


(stanleykipkoech8@gmail.com)
lOMoARcPSD|3982611

CHAPTER 2. LIFE TABLES AND SELECTION 15

The first formula states that the number of individuals alive at age x is equal to the total number of
deaths in each of the remaining years, the second states that the number of deaths that occur in the
next t years is equal to the sum of the number of deaths in each of the following t years.

Example: Consider the following life table


x lx
30 10000
.. ..
. .
35 9789.29
36 9734.12
37 9673.56
38 9607.07
39 9534.80

Calculate the following probabilities


(i) The probability that an individual aged 30 will survive for another 5 years.
(ii) The probability that an individual aged 30 will die between the ages of 36 and 38.

Solution: For the first part we have


l35 9789.29
5 p30 = = = 0.978929
l30 10000
For the second part we require a deferred probability so we need to use either equation (1.13) or (1.14).
Using equation (1.13) we find that

l36 l38 9734.12 9607.07


6|2 q30 = 6 p30 − 8 p30 = − = − = 0.012705
l30 l30 10000 10000

The force of mortality can also be calculated using life tables. Recall from chapter 1 that the force of
mortality may be written as
d 1 d t px
µx+t = − log (t px ) = −
dt t px dt

Using equation (2.2) and noticing that lx is constant under differentiation with respect to t,

lx d lx+t
µx+t = −
lx+t dt lx
1 dlx+t
=−
lx+t dt
As with the force of mortality, the equations for the expected future lifetime remain the same as in
chapter 1 with our life table expression of the survival function substituted accordingly. The complete
expectation of life may therefore be written as
Z ∞ Z ∞
lx+t
e̊x = t px dt = dt
0 0 lx
1 ∞
Z
= lx+t dt
lx 0
1 ∞
Z
= lz dz
lx x
where the change of variable z = x + t is used in the final line.

Downloaded by Kipkoech Stanley (stanleykipkoech8@gmail.com)


lOMoARcPSD|3982611

CHAPTER 2. LIFE TABLES AND SELECTION 16

Similarly, the curtate expectation of life is given by


∞ ∞
X X lx+k
ex = k px =
k=1 k=1
lx

Example: Consider the following survival model


1000
lx = x≥0
(1 + x)2

Derive an expression for the mean of Tx .

Solution: We first need to find an expression for the survival function t px . Using equation (2.2) we
have
lx+t (1 + x)2
p
t x = =
lx (1 + x + t)2
The mean of Tx is then given by
∞ ∞
(1 + x)2
Z Z
e̊x = p
t x dt = dt
0 0 (1 + x + t)2
Z ∞
2
u=∞
u−2 du = −(1 + x)2 u−1 u=1+x

= (1 + x)
1+x
(1 + x)2
 
=0− −
1+x
=1+x

Example: Consider the extract of a life table provided below


x lx
94 16208
95 10902
96 7212
97 4637
98 2893
99 1747
100 0

Evaluate the following


(i) e95
(ii) the variance of K95
(iii) e95:3

Solution: For the first part we use the equation for the curtate expectation of life, it is only necessary
to sum over the first 4 terms as these are the only non-zero terms given the age of the individual.
∞ 4
X X l95+k
e95 = k p95 =
k=1 k=1
l95
l96 + l97 + l98 + l99
=
l95
7212 + 4637 + 2893 + 1747
=
10902
= 1.5125

Downloaded by Kipkoech Stanley (stanleykipkoech8@gmail.com)


lOMoARcPSD|3982611

CHAPTER 2. LIFE TABLES AND SELECTION 17

To calculate the variance of K95 we first need to find the second moment E[Kx2 ]. For this we do not
have a simple equation as we do for the first moment so we instead use the definition of the second
moment for a discrete random variable.

X ∞
X 4
X
2 2 2
E[K95 ] = k Pr(K95 = k) = k k| q95 = k 2 (k p95 − k+1 p95 )
k=0 k=0 k=0
4  
X l95+k − l95+k+1
= k2
k=0
l95
l96 − l97 16(l99 − l100 )
=0+ + ... +
l95 l95
7212 16(1747 − 0)
= 10902 + ... +
4637 10902

= 4.3861

The variance can now be calculated as follows


2
Var[K95 ] = E[K95 ] − (e95 )2 = 4.3861 − (1.5125)2 = 2.0984

Finally, the temporary curtate expectation of life can be calculated in the same way as the curtate
expectation of life from part (i) with the upper limit of the sum now set equal to 3.
3 3
X X l95+k
e95:3 = k p95 =
k=1 k=1
l95
l96 + l97 + l98
=
l95
7212 + 4637 + 2893
=
10902
= 1.3522

2.2 Uniform distribution of deaths


Life tables only show values of lx for x a non-negative integer, however, we may require mortality
probabilities for non-integer ages. In this case we can estimate the values of lx using fractional age
assumptions, the two assumptions we consider in this course are the uniform distribution of deaths and
constant force of mortality.
The UDD assumption is the simplest of the fractional age assumptions and assumes that between
integer-valued years, the death rate is constant. Suppose we have lx > lx+t > lx+1 with x < x+t < x+1,
since we are assuming a constant death rate, linear interpolation can be used to obtain an expression
for lx+t .

(lx+t − lx+1 )(x + 1 − x) = (lx − lx+1 )(x + 1 − (x + t))

=⇒ (lx+t − lx+1 ) = (lx − lx+1 )(1 − t)

=⇒ lx+t = lx − t(lx − lx+1 )

=⇒ lx+t = lx − tdx (2.6)

Downloaded by Kipkoech Stanley (stanleykipkoech8@gmail.com)


lOMoARcPSD|3982611

CHAPTER 2. LIFE TABLES AND SELECTION 18

Equation (2.6) is one of the most important equations in this section as it allows us to derive expressions
for the survival, distribution, and density functions, and the force of mortality under the UDD
assumption. For integer x and 0 ≤ t ≤ 1 we have
lx+t lx − tdx tdx
t px = = =1− = 1 − tqx using equation (2.5)
lx lx lx

d t px d
fx (t) = − = − (1 − tqx ) = qx (2.7)
dt dt
f0 (x) qx
µx+t = =
t px 1 − tqx

t qx = 1 − tpx = tqx (2.8)

It is not necessary to learn all four of the expressions above since by knowing any one of them the others
may be easily determined. The most useful expression is that for the distribution function as this is the
simplest to apply when calculating probabilities.

Example: You are given the following extract from a life table

x lx
35 97250
36 97126
37 96993

Under the UDD assumption, compute the probabilities (i) 0.5 p35 , (ii) 1.5 p35

Solution: For the first part we have


l36 97126
0.5 p35 = 1 − 0.5 q35 = 1 − 0.5q35 where q35 = 1 − p35 = 1 − =1− = 0.001275
l35 97250
Therefore,
0.5 p35 = 1 − (0.5 · 0.001275) = 0.9993625
For the second part we need to be careful as we cannot immediately apply equation (2.8). This equation
can only be used when 0 ≤ t ≤ 1 but in this case we have t = 1.5. We must instead first use the
multiplicative property of the survival function to write the probability as

1.5 p35 = 0.5 p36 · p35

The UDD assumption can now be applied to evaluate 0.5 p36 while p35 can be calculated using standard
results.
   
l37 96993
0.5 p36 = 1 − 0.5 q36 = 1 − 0.5q36 = 1 − 0.5 1 − = 1 − 0.5 1 − = 0.9993
l36 97126

l36 97126
p35 = = = 0.9987
l35 97250
The overall probability is therefore

1.5 p35 = 0.9993 · 0.9987 = 0.9980

Downloaded by Kipkoech Stanley (stanleykipkoech8@gmail.com)


lOMoARcPSD|3982611

CHAPTER 2. LIFE TABLES AND SELECTION 19

2.3 Constant force of mortality


The assumption of a constant force of mortality is based on the idea of exponential interpolation and
assumes that lx+t = abt , 0 ≤ t ≤ 1. We can determine the values of a and b by considering different
values of t.
t = 0 =⇒ lx = a
lx+1 lx+1
t = 1 =⇒ lx+1 = ab =⇒ b = =
a lx
Therefore
 t
lx+1
lx+t = lx = (lx )1−t (lx+1 )t (2.9)
lx

As with equation (2.6) in the previous lecture, equation (2.9) is one of the key equations for the constant
force of mortality assumption and we now use it to derive expressions for the survival function, density
function, and force of mortality.
t
(lx )1−t (lx+1 )t

lx+t lx+1
t px = = = =⇒ t px = (px )t (2.10)
lx lx lx

=⇒ t qx = 1 − (px )t

To find the p.d.f we recall that when differentiating ax with respect to x we instead apply the chain
rule to exp(xln(a)), therefore,

d t px d d
fx (t) = − = − (px )t = − exp(tlog(px )) = −ptx log(px )
dt dt dt
Finally, the force of mortality is given by

fx (t) pt log(px )
µx+t = =− x t = −log(px )
t px px

Note that since 0 ≤ px ≤ 1, log(px ) ≤ 0 and so the force of mortality and p.d.f are non-negative as
required.

Example: You are given the following extract from a life table.
x lx
80 53925
81 50987
82 47940
83 44803

Under the assumption of a constant force of mortality between integer ages, calculate the probabilities
(i) 0.75 p80 , (ii) 2.25 p80

Solution: Assuming a constant force of mortality, the most useful equation to calculate probabilities
is (2.10). Since for the first part t ≤ 1, this equation can be applied straight away to give
 0.75  0.75
0.75 l81 50987
0.75 p80 = (p80 ) = = = 0.95885
l80 53925

Downloaded by Kipkoech Stanley (stanleykipkoech8@gmail.com)


lOMoARcPSD|3982611

CHAPTER 2. LIFE TABLES AND SELECTION 20

For the second part we use the multiplicative property of the survival function to give

2.25 p80 = 2 p80 · 0.25 p82

where
l82 47940
2 p80 = = = 0.8890
l80 53925
and  0.25  0.25
0.25 l83 44803
0.25 p82 = (p82 ) = = = 0.9832.
l82 47940

The total probability is therefore

2.25 p80 = 0.8890 · 0.9832 = 0.8741.

2.4 Select and ultimate life tables


When an individual purchases an insurance contract from a company, the company will choose a
premium from a schedule of rates that depend on the size of the policy and other factors known
as rating factors. This allows the company to assess the risks for each life by considering such factors as
their age, sex, smoking habits, and family health history. Suppose that an individual passes a series of
health checks set out by the insurance company, the individual is said to be select and lower mortality
rates will apply. Since the health of the individual could quickly deteriorate, the life usually remains
select for only a couple of years, a period known as the select period. When the select period is over
the life is referred to as ultimate. The age that a life is selected at is denoted by [x] and if the life is still
alive a year later and still in the select period, the notation [x]+1 is used to denote their age. When
the life is no longer in the select period we drop the square brackets and revert back to the original
notation. Using this notation, the survival function for a life select at age x is t p[x] .

Example: Consider the following extract from a select life table


x q[x] q[x]+1 q[x]+2
35 0.013 0.012 0.011
If l[35] = 1000, find l[35]+1 and l[35]+2 .

Solution: We know
l[35]+1
p[35] = =⇒ l[35]+1 = p[35] · l[35] = (1 − q[35] ) · l[35] = (1 − 0.013) · 1000 = 987
l[35]

and hence
l[35]+2
p[35]+1 = =⇒ l[35]+2 = p[35]+1 · l[35]+1 = (1 − q[35]+1 ) · l[35]+1 = (1 − 0.012) · 987 = 975.16
l[35]+1

Example: Consider the following two-year select life table


x l[x] l[x]+1 lx+2 x+2
45 1235 1124 1039 47
46 1135 1025 978 48
47 1012 996 965 49
Calculate (i) 2 p[46]+1 and (ii) 0.4 p[45]+1.7 , using the UDD assumption for the second part.

Downloaded by Kipkoech Stanley (stanleykipkoech8@gmail.com)


lOMoARcPSD|3982611

CHAPTER 2. LIFE TABLES AND SELECTION 21

Solution If we notice that since the select period is 2 years, after 3 years the life will no longer be select
and l[x]+3 = lx+3 , then
l[46]+3 l49 965
2 p[46]+1 = = = = 0.9415
l[46]+1 l[46]+1 1025
For the second part
l[45]+2.1 l47.1
0.4 p[45]+1.7 = =
l[45]+1.7 l([45]+1)+0.7

Now, using equation (2.6) and considering the numerator and denominator separately

l47.1 = l47 − 0.1(l47 − l48 ) = 1039 − 0.1(1039 − 978) = 1032.9

and
l([45]+1)+0.7 = l[45]+1 − 0.7(l[45]+1 − l[45]+2 ) = l[45]+1 − 0.7(l[45]+1 − l47 )
= 1124 − 0.7(1124 − 1039)
= 1064.5

Therefore, the overall probability is


1032.9
0.4 p[45]+1.7 = = 0.9703
1064.5

Example: A select and ultimate life table has a 3 year select period. You are given the following
information
1
• q[x] = 6

• lx+6 = 90000
4
• 5 p[x+1] = 5
9
• 3 p[x]+1 = p
10 3 [x+1]

Evaluate l[x]

1
Solution: We can start by considering which equations involve lx , since we know q[x] = 6
we can easily
determine that p[x] = 56 and so the most obvious choice is

l[x]+1 l[x]+1 6
p[x] = =⇒ l[x] = = · l[x]+1
l[x] p[x] 5

We therefore need to find an expression for l[x]+1 . Using the final bullet point, we have

l[x]+4 9 l[x+1]+3
= ·
l[x]+1 10 l[x+1]

But since the select period is 3 years, l[x]+4 = l[x+1]+3 = lx+4 , and these terms cancel on each side.
This leaves
1 9 1 10
= · =⇒ l[x]+1 = · l[x+1]
l[x]+1 10 l[x+1] 9
We now need to compute the value of l[x+1] which may be done using the remaining two bullet points

lx+6 lx+6 5
5 p[x+1] = =⇒ l[x+1] = = 90000 · = 112500
l[x+1] 5 p[x+1] 4

Downloaded by Kipkoech Stanley (stanleykipkoech8@gmail.com)


lOMoARcPSD|3982611

CHAPTER 2. LIFE TABLES AND SELECTION 22

and therefore
10
l[x]+1 = · 112500 = 125000
9
Finally, our expression for l[x] is
6
l[x] = · 125000 = 150000
5

Downloaded by Kipkoech Stanley (stanleykipkoech8@gmail.com)


lOMoARcPSD|3982611

Chapter 3

Life Insurance Benefits

3.1 Introduction to life insurance functions


In this chapter we develop formulae for the expected present value of the benefits paid by the insurance
company for a number of different contract types. We use the future lifetime random variable Tx to
determine the value of a benefit payable immediately on the death of an individual, and the curtate
future lifetime random variable for when a payment is made at the end of an agreed period, for example,
at the end of the month of death. Relationships are then developed between the different insurance
benefits and it is shown how they may be combined in the case of more complex insurance policies.
Before we define any of the functions we give a brief introduction into the traditional types of life
insurance policy.

Term insurance: A term insurance pays a benefit on the death of the policyholder provided they
die within a period of n years as specified in the contract, no payment is made if the life survives the
duration of the term. Premiums are often paid at the start of each year for the duration of the term.

Whole life insurance: A whole life insurance pays a benefit on the death of the policyholder whenever
this may occur. Premiums are generally paid at the start of each year given the individual is alive,
however, sometimes they are payable up to some maximum age to avoid the problems older lives might
face in paying them.

Pure Endowment insurance: A pure endowment insurance pays a benefit to the policyholder at the
end of an n year period provided they are still alive, similar to a term insurance, premiums are paid
annually for the duration of the term.

Endowment insurance: An endowment insurance is a combination of a term and a pure endowment


insurance. A benefit is payable either on the death of the policyholder or at the end of an n year term,
whichever occurs first. Premiums are again generally paid at the start of each year for the duration of
the term but since a benefit payment is guaranteed, the premiums for this type of policy tend to be
significantly higher.

The following results should be familiar from previous courses in financial mathematics but are provided
anyway. Given an annual effective interest rate i, the annual interest rate convertible mthly satisfies
m
i(m)

1+i= 1+
m

and if the interest is compounded continuously we use the force of interest which is given by

δ = log(1 + i)

Downloaded by Kipkoech Stanley23


(stanleykipkoech8@gmail.com)
lOMoARcPSD|3982611

CHAPTER 3. LIFE INSURANCE BENEFITS 24

The annual effective discount rate can be defined in terms of the discount factor v as
1
d = iv = 1 − v where v= = e−δ
1+i
Finally, given d, the annual discount rate convertible mthly satisfies
m
d(m)

1−d= 1−
m

3.2 Term and Whole life insurance


We can now begin to define the expected present value (EPV) of the benefits for each type of insurance
policy, starting with a term insurance. For all the insurance functions we use the present value random
variable Z which denotes the value of a unit benefit payment at the time of issue.

Term insurance (continuous case)


Consider an n year term insurance with unit benefit payable immediately on the death of an individual.
If the individual is currently aged x, the benefit will be paid in Tx years given Tx ≤ n, therefore, to
get the present value of a unit benefit the payment must be discounted by Tx years. The present value
random variable may then be written as

v x = e−δTx
 T
if Tx ≤ n
Z=
0 if Tx > n
The expected value of Z, sometimes referred to as the actuarial present value (APV) and denoted by
1
Āx:n , is
Z n Z n
1 −δt
Āx:n = E[Z] = e fx (t) dt = e−δt t px µx+t dt (3.1)
0 0

In actuarial notation, the bar above the A denotes that the benefit is payable immediately on death
and the ‘1’ above the x indicates that for the benefit to be paid the life must die before the end of the
n year term. To find the second moment of the random variable Z, we consider the rule of moments.
Suppose we want to find the j th moment, we would use the following formula
Z n Z n Z n
−δt j −(δj)t
j
e−γt fx (t) dt

E[Z ] = e fx (t) dt = e fx (t) dt =
0 0 0

where γ = δj can be thought of as a new force of interest. Since the final equality gives our formula for
the expectation of Z, we have a simple rule for finding the second moment.

E[Z 2 ] δ = E[Z]|2δ

(3.2)

That is, if E[Z] is evaluated with a force of interest δ, E[Z 2 ] is equal to E[Z] evaluated with a force
of interest equal to 2δ. For a term insurance the second moment of Z is denoted by 2 Āx:n 1
and so the
formula for calculating the variance is
2
Var[Z] = 2 Āx:n
1 1
− Āx:n (3.3)

Example: A survival model is defined with a constant force of mortality µx+t = µ. For an n year
term insurance find the EPV of a unit benefit payable immediately on death and the variance of the
present value random variable.

Downloaded by Kipkoech Stanley (stanleykipkoech8@gmail.com)


lOMoARcPSD|3982611

CHAPTER 3. LIFE INSURANCE BENEFITS 25

Solution: To find the EPV of the benefit we need the survival function given a constant force of
mortality. Using results from the first chapter it can be shown that

t px = e−µt

Substituting this expression into equation (3.1) gives


Z n Z n
1 −δt −µt
Āx:n = e µe dt = µe−(µ+δ)t dt
0 0
µ  −(µ+δ)t t=n
=− e t=0
µ+δ
µ
1 − e−(µ+δ)n

=
µ+δ
To find the variance of the present value random variable we need to calculate the second moment.
1
Using the rule of moments we simply need to replace every δ in our expression for Āx:n with 2δ, this
gives
2 1 µ
1 − e−(µ+2δ)n

Āx:n =
µ + 2δ
Therefore, the expression for the variance is
 2
2 1 1
2 µ µ
1 − e−(µ+2δ)n − 1 − e−(µ+δ)n
 
Var[Z] = Āx:n − Āx:n =
µ + 2δ µ+δ

Example: An individual currently aged 40 purchases a 15-year term insurance that pays a benefit of
£50,000 immediately on death. If the force of interest is δ = 0.05 and
x
S0 (x) = 1 − 0 ≤ x ≤ 100
100

(i) Find the EPV of the death benefit.


(ii) Find the standard deviation of the present value random variable.

Solution: Using equation (1.10), the survival function is

S0 (x + t) 100 − x − t 100 t
t px = = · =1−
S0 (x) 100 100 − x 100 − x

Hence for a life aged 40,


t
t p40 =1−
60

Since we have not been given the force of mortality, it is much easier to compute the EPV of the benefit
using the formula involving the density, where
d 1
f40 (t) = − t p40 =
dt 60
The EPV of a unit benefit is therefore
15 15
1
Z Z
1 −δt
A40:15 = e f40 (t) dt = e−δt dt
0 0 60
1  −δt 15
=− e 0
60δ
1
1 − e−15δ = 0.1759

=
60δ

Downloaded by Kipkoech Stanley (stanleykipkoech8@gmail.com)


lOMoARcPSD|3982611

CHAPTER 3. LIFE INSURANCE BENEFITS 26

However, since the policy pays £50,000 on death, the EPV of the benefit is
1
£50, 000Āx:n = £50, 000 × 0.1759 = £8, 793.89

Using the rule of moments, the second moment of the present value random variable is
1
E[Z 2 ] = 1 − e−30δ = 0.1295

120δ
When finding the variance we need to make sure we find it for the total benefit and not just a unit
benefit
Var[50000Z] = 500002 Var[Z] = 500002 2 Ā40:15
1 1
)2

− (Ā40:15
= 500002 (0.1295 − 0.17592 )
= 246, 397, 975

Whole life insurance (continuous case)


For a whole life insurance with a unit benefit payable immediately on death there is no condition on
when the death must occur, therefore, for an individual currently aged x the benefit will be paid in Tx
years. The present value random variable may then be defined as

Z = v Tx = e−δTx

The expected value of Z is denoted in actuarial notation by Āx and is given by the following formula
Z ∞ Z ∞
−δt
Āx = e fx (t) dt = e−δt t px µx+t dt (3.4)
0 0

The variance of Z is
Var[Z] = 2 Āx − (Āx )2

where the second moment can again be calculated using the rule of moments. In fact, for all the insurance
functions considered in this course the rule of moments can be applied to find higher moments, this will
not be the case with the annuity functions considered in chapter 4.

Example: A survival model has constant force of mortality µx+t = 0.01. A life currently aged 30
purchases a whole life insurance policy with sum insured £1. Find the EPV of the benefit if the force
of interest is δ = 0.05.

Solution: We could solve this question by using equation (3.4) and evaluating the integral, however,
there is an alternative method using a result from a previous example. In the first term insurance
example the survival model also had a constant force of mortality and we found that for the general
case
1 µ
1 − e−(µ+δ)n

Āx:n =
µ+δ
A whole life insurance is simply the limiting case of a term insurance as n → ∞, therefore, to find Āx
we only need to consider the limit as n → ∞ of the equation above.
1 µ
=⇒ Āx = lim Āx:n =
n→∞ µ+δ

Downloaded by Kipkoech Stanley (stanleykipkoech8@gmail.com)


lOMoARcPSD|3982611

CHAPTER 3. LIFE INSURANCE BENEFITS 27

Substituting for the values of µ and δ as given in the question gives


0.01
Āx = = 0.1667
0.01 + 0.05

3.3 Endowment and deferred insurances


Pure endowment insurance
For an n-year pure endowment insurance a benefit is paid if the individual dies after n years, that is
Tx ≥ n. Since the benefit is only ever paid after n years, the present value random variable is

0 if Tx < n
Z=
v n = e−δn if Tx ≥ n

Z is a discrete random variable that takes only two values, 0 with probability n qx , and v n with probability
n px , therefore, the expected value is given by

Ax:n1 = n Ex = v n n px (3.5)

The ‘1’ over the n indicates that for the benefit to be paid the term must expire before the life and there
is no bar over the A as there only exists a discrete time version for a pure endowment insurance. This
notation is not favoured due to its similarity to the notation for the EPV of a term insurance benefit,
so for this reason the more convenient notation n Ex is used instead. The second moment of Z can be
calculated in the same way as for any discrete random variable
2
Ax:n1 = (v n )2 · n px = v 2n n px

and therefore, the variance is


2
Var[Z] = 2 Ax:n1 − Ax:n1

Endowment insurance (continuous case)


With an n-year endowment insurance, if the life dies during the term the benefit is paid immediately on
death, if the life survives, the benefit is paid at the end of the term. The present value random variable
is

v x = e−δTx
 T
if Tx < n
Z= n −δn
v =e if Tx ≥ n

= v min(Tx ,n) = e−δmin(Tx ,n)


This may be written as the sum of the present value random variables for a term and pure endowment
insurance.
 T 
v x if Tx < n 0 if Tx < n
Z = + n
0 if Tx ≥ n v if Tx ≥ n

Therefore, the expectation of Z is equal to the sum of the expectations of the term and pure endowment
present value random variables, that is
1
Āx:n = Āx:n + n Ex (3.6)

Downloaded by Kipkoech Stanley (stanleykipkoech8@gmail.com)


lOMoARcPSD|3982611

CHAPTER 3. LIFE INSURANCE BENEFITS 28

Similarly, the expected value of Z 2 is given by


2
Āx:n = 2 Āx:n
1
+ v 2n n px

Deferred insurance (continuous case)


An n-year deferred insurance policy pays a benefit on death given the insured dies at least n years after
issue, no benefit is paid if the life dies during the first n years, known as the deferred period. The
present value random variable is

0 if Tx < n
Z= Tx −δTx
v =e if Tx ≥ n

The actuarial notation for the EPV of a deferred insurance benefit is


Z ∞ Z ∞
−δt
n| Āx = e fx (t) dt = e−δt t px µx+t dt (3.7)
n n

The variance of Z is 2
Var[Z] = 2 n| Āx − n| Āx

where the rule of moments can be used to calculate the expectation of the squared present value.

3.4 Insurance functions for De Moivre’s model


The following results are an application of the formulae defined in the previous two lectures to the De
Moivre model, you should be familiar with the results and the derivations. All of the results involve
continuous annuities certain so let us recall from previous financial mathematics courses that
Z n
1 − e−δn
ān = e−δt dt =
0 δ

Remember also that since there is always a limiting age with the De Moivre model, we should never
have the upper limit of an integral set equal to infinity. If a life is currently aged x and the limiting
age is ω, they can only survive for a maximum of ω − x years so this would be the upper limit of the
integral.

Theorem: For De Moivre’s model with terminal age ω and n ≤ ω − x

āω−x 1 ān e−δn āω−x−n


(i) Āx = (ii) Āx:n = (iii) n| Āx =
ω−x ω−x ω−x

1
Proof (i): For De Moivre’s model it can be shown that fx (t) = ω−x
, using this along with equation

Downloaded by Kipkoech Stanley (stanleykipkoech8@gmail.com)


lOMoARcPSD|3982611

CHAPTER 3. LIFE INSURANCE BENEFITS 29

(3.4) gives
∞ ω−x
1
Z Z
−δt
Āx = e fx (t) dt = e−δt dt
0 0 ω−x
1  −δt t=ω−x
=− e t=0
δ(ω − x)
1 − e−δ(ω−x)
=
δ(ω − x)
āω−x
=
ω−x


(ii) Starting with the definition of the EPV of a unit benefit for a term insurance
Z n Z n
1 −δt 1 1 − e−δn
Āx:n = e fx (t) dt = e−δt dt =
0 0 ω−x δ(ω − x)
1 t=n
e−δt t=0
 
=−
δ(ω − x)
ān
=
ω−x


(iii) Using equation (3.7)


Z ∞ Z ω−x
−δt 1
n| Āx = e fx (t) dt = e−δt dt
n n ω−x
Z ω−x−n
1
= e−δ(s+n) ds (using the substitution t = s + n)
0 ω−x
Z ω−x−n
e−δn
= e−δs ds
ω−x 0
e−δn āω−x−n
=
ω−x


Properties of insurance functions


The following two relationships link the insurance functions for the different types of policy, they are
useful when less traditional contracts are involved and need to be defined using the sum of multiple
insurances. The first relationship is fairly intuitive and links whole life, term, and deferred insurances.
1
Āx = Āx:n + n| Āx (3.8)

Proof: Z n Z ∞ Z ∞
1 −δt −δt
Āx:n + n| Āx = e fx (t) dt + e fx (t) dt = e−δt fx (t) dt = Āx
0 n 0

This relationship also holds for higher moments, that is


2
Āx = 2 Āx:n
1
+ 2n| Āx (3.9)

Downloaded by Kipkoech Stanley (stanleykipkoech8@gmail.com)


lOMoARcPSD|3982611

CHAPTER 3. LIFE INSURANCE BENEFITS 30

The second relationship links whole life, deferred, and pure endowment insurances

n| Āx = n Ex Āx+n (3.10)

Proof:
Z ∞ Z ∞
−δt
n| Āx = e fx (t) dt = e−δt t px µx+t dt
Zn ∞ n

= e−δ(s+n) s+n px µx+s+n ds (using the substitution t = s + n)


0
Z ∞
−δn
= e n px e−δs s px+n µx+s+n ds = n Ex Āx+n
0


3.5 Examples - continuous insurances


We begin with a number of examples that demonstrate how the relationships defined in the previous
lecture may be used to calculate the EPV of benefits for more complex insurance policies.

Example: Consider a whole life insurance with a benefit of £1000 payable immediately on death, the
force of interest and force of mortality are
 
0.04 0 ≤ t ≤ 10 0.006 0 ≤ t ≤ 10
δ= , µ=
0.05 t > 10 0.007 t > 10

Calculate the EPV of the benefit.

Solution: Since the force of interest and force of mortality change after 10 years, we need to consider
the EPV in two separate parts, from t = 0 to t = 10 and then from t = 10 onwards. For the first
part a benefit is paid provided the life dies within the first 10 years, this is how we define a 10-year
1
term insurance so the EPV is given by Āx:10 . For the second part a benefit is paid on death given the
life survives for the first 10 years, this is how we define a deferred life insurance so the EPV is 10| Āx .
Therefore, the total EPV is
1
Ax = Āx:10 + 10| Āx

which is also just an application of equation (3.8) with n = 10.

We now need to evaluate each of the insurance functions using the appopriate values for the force of
mortality and force of interest. Note that in both cases the force of mortality is constant so the density
function is given by fx (t) = µe−µt . For the term insurance we have

Z 10 Z 10
1 −δt −µt µ  −(δ+µ)t t=10
Āx:10 = e µe dt = µ e−(δ+µ)t dt = − e t=0
0 0 µ+δ
µ
1 − e−10(δ+µ)

=
µ+δ
0.006
1 − e−0.46

=
0.046

= 0.04809

Downloaded by Kipkoech Stanley (stanleykipkoech8@gmail.com)


lOMoARcPSD|3982611

CHAPTER 3. LIFE INSURANCE BENEFITS 31

and for the deferred insurance


Z ∞
µ  −(δ+µ)t t=∞
10| Āx = µ e−(δ+µ)t dt = − e t=10
10 µ+δ
µ
e−10(δ+µ)

=
µ+δ
0.007 −0.57 
= e
0.057

= 0.06945

Therefore, the total EPV for a benefit of £1000 is

£1000Ax = £1000(0.04809 + 0.06945) = $117.54.

Example: A life currently aged 40 purchases a whole life insurance policy that pays £10 should death
occur in the first 10 years and £5 in subsequent years, in both cases the benefits are paid immediately
on death. De Moivre’s model is used with limiting age ω = 100 and the force of interest is δ = 0.05.
Find the EPV of the death benefit.

Solution: As with the previous example we need to consider the EPV from t = 0 to t = 10 and then
t = 10 onwards, however, in this case there are two approaches that can be taken. The first method
treats the insurance as the sum of a 10-year term insurance paying £10 and a 10-year deferred insurance
paying £5 whereas for the second method we think of the insurance as the sum of a whole life insurance
paying £5 and a 10-year term insurance also paying £5. Graphs showing how each method splits up
the benefit are given below, the first method is to the left and the second is on the right.

Using equation (3.8) it may be shown that the two methods are equivalent
1 1 1 1

5Ā40 + 5Ā40:10 = 5 Ā40:10 + 10| Ā40 + 5Ā40:10 = 10Ā40:10 + 510| Ā40

Therefore, which method you use depends on whether you find it easier to visualise dividing the benefit
according to age or the amount of benefit.
1
The density function for De Moivre’s model with a life aged 40 is fx (t) = 60 , so

Downloaded by Kipkoech Stanley (stanleykipkoech8@gmail.com)


lOMoARcPSD|3982611

CHAPTER 3. LIFE INSURANCE BENEFITS 32

60
1 1  −δt t=60 1
Z
e−δt −3

Ā40 = dt = − e t=0
= 1 − e = 0.31674
0 60 60δ 3

10
1 1  −δt t=10 1
Z
1
e−δt −0.5

Ā40:10 = dt = − e t=0
= 1 − e = 0.13116
0 60 60δ 3

60
1 1  −δt t=60 1 −0.5
Z
e−δt −3

10| Ā40 = dt = − e t=10
= e − e = 0.18558
10 60 60δ 3

For the first method we have


1
EPV = 10Ā40:10 + 510| Ā40 = (10 · 0.131156) + (5 · 0.202177) = 2.2395

and for the second method


1 1
EPV = 5Ā40 + 5Ā40:10 = (5 · ) + (5 · 0.131156) = 2.2395
3

3.6 Discrete time insurance functions


When a policy pays a benefit at the end of the year of death the present value random variable depends
on the curtate future lifetime random variable Kx . We now define discrete time insurance functions
analogous to those defined in the previous lectures for continuous time.

Whole life insurance (discrete case)


Recall that Kx represents the whole number of years an individual survives, if a benefit is paid at the
end of the year of death the benefit will be paid at Kx + 1. The present value random variable is
therefore given by
Z = v Kx +1

The actuarial notation for the expected value of Z is



X ∞
X
k+1
Ax = v Pr(Kx = k) = v k+1 k| qx (3.11)
k=0 k=0

As before, we can find higher moments using the rule of moments but since for discrete insurances we
use the annual effective interest rate rather than the force of interest, the rule differs slightly. Starting
from the definition of the j th moment, we want to find a new rate α that allows us to write the higher
moments in terms of the first. The j th moment of Z is given by
∞ ∞
k+1 j
X X
j
v j(k+1) k| qx

E[Z ] = v k| qx =
k=0 k=0

Downloaded by Kipkoech Stanley (stanleykipkoech8@gmail.com)


lOMoARcPSD|3982611

CHAPTER 3. LIFE INSURANCE BENEFITS 33

We need to find a rate α such that


1 1
v j(k+1) i = v k+1 α ⇐⇒ j(k+1)
=
(1 + i) (1 + α)k+1
 k+1
1+α
⇐⇒ =1
(1 + i)j
1+α
⇐⇒ =1
(1 + i)j
⇐⇒ α = (1 + i)j − 1

Therefore, for discrete insurances using an effective rate of interest i, the rule of moments states that
the second moment satisfies
E[Z 2 ] i = E[Z]|(1+i)2 −1

(3.12)

The variance for a discrete whole life insurance is given by

Var[Z] = 2 Ax − (Ax )2 (3.13)

Term insurance (discrete case)


For an n-year term insurance with unit benefit payable at the end of the year of death, the present
value random variable is  K +1
v x if Kx ≤ n − 1
Z=
0 if Kx ≥ n
and the expected value is given by
n−1
X
1
Ax:n = v k+1 k| qx (3.14)
k=0

Note that if a life dies in the last year of the term, the payment will be made at t = n but Kx = n − 1,
hence why the upper limit of the sum is n − 1. The variance of Z is
2
Var[Z] = 2 Ax:n
1 1
− Ax:n (3.15)

Example: A survival model is defined with constant force of mortality µx+t = µ, show that the EPV
of a unit benefit for a whole life insurance may be written as
eµ − 1
Ax =
eµ+δ − 1
Solution: Using the defintion of Ax

X ∞
X
k+1
Ax = v k| qx = v k+1 k px qx+k
k=0 k=0

For a constant force of mortality we know that the survival function is given by k px = e−µk and since
this is independent of age, it follows that px+k = e−µ and hence qx+k = 1 − e−µ . Substituting these

Downloaded by Kipkoech Stanley (stanleykipkoech8@gmail.com)


lOMoARcPSD|3982611

CHAPTER 3. LIFE INSURANCE BENEFITS 34

expressions into the previous equation and writing the discount factor in exponential form gives

X
−µ
Ax = (1 − e ) e−δ(k+1) e−µk
k=0

X
−δ −µ
= e (1 − e ) (e−(δ+µ) )k
k=0
−δ −µ
e (1 − e )
=
1 − e−(δ+µ)
eµ − 1
= (δ+µ)
e −1
where the following formula for an infinite geometric series has been used to evaluate the sum

X a
ark =
k=0
1−r

Endowment insurance (discrete case)


For an n-year endowment insurance with unit benefit payable at the end of the year of death, the
present value random variable is
 K +1
v x if Kx ≤ n − 1
Z=
vn if Kx ≥ n

As with the continuous case, the EPV is equal to the sum of the EPV for a term and pure endowment
insurance benefit, that is

1
Ax:n = Ax:n + n Ex (3.16)
The variance of Z is then given by

Var[Z] = 2 Ax:n − (Ax:n )2 (where 2


Ax:n = 2 Ax:n
1
+ 2 Ax:n1 ) (3.17)

Deferred insurance (discrete case)


For an n-year deferred insurance paying a unit benefit at the end of the year of death, the present value
random variable is 
0 if Kx ≤ n − 1
Z=
v Kx +1 if Kx ≥ n

The expected value of Z is given in actuarial notation by



X
n| Ax = v k+1 k| qx (3.18)
k=n

The variance of Z is 2
Var[Z] = 2n| Ax − n| Ax (3.19)

The two relationships that we previously derived for continuous insurance functions also apply in the
discrete case, that is
1
Ax = Ax:n + n| Ax (3.20)

Downloaded by Kipkoech Stanley (stanleykipkoech8@gmail.com)


lOMoARcPSD|3982611

CHAPTER 3. LIFE INSURANCE BENEFITS 35

Proof:
n−1
X ∞
X ∞
X
1 k+1 k+1
Ax:n + n| Ax = v k| qx + v k| qx = v k+1 k| qx = Ax
k=0 k=n k=0


n| Ax = n Ex Ax+n (3.21)
Proof:

X ∞
X
k+1
n| Ax = v k| qx = v j+n+1 j+n| qx
k=n j=0

X ∞
X
n j+1 n
=v v j+n px · qx+j+n = v v j+1 j px+n · n px · qx+j+n
j=0 j=0

X
= v n n px v j+1 j px+n · qx+j+n
j=0

= n Ex Ax+n


3.7 Recursive Relationships


The life tables introduced in chapter 2 are not only used to display the number of individuals still alive
or the mortality probabilites, sometimes the tables also provide values of the insurance functions for
varying ages. When producing these tables, instead of calculating the value of the function for each age,
we can do so for a single age and then use a recursive formula. We now state and prove the recursive
formulae for the whole life, term, deferred, and endowment insurance functions.

Ax = vqx + vpx Ax+1 (3.22)


Proof:

X ∞
X
k+1
Ax = v k| qx = v k+1 k px · qx+k
k=0 k=0

X
= vqx + v k+1 k px · qx+k
k=1

X
= vqx + vpx v k k−1 px+1 · qx+k
k=1

X
= vqx + vpx v j+1 j px+1 qx+j+1 (using the substitution j = k − 1)
j=0
X∞
= vqx + vpx v j+1 j| qx+1 = vqx + vpx Ax+1
j=0

1 1
Ax:n = vqx + vpx Ax+1:n−1 (3.23)

Downloaded by Kipkoech Stanley (stanleykipkoech8@gmail.com)


lOMoARcPSD|3982611

CHAPTER 3. LIFE INSURANCE BENEFITS 36

Proof:
n−1
X n−1
X
1 k+1
Ax:n = v k| qx = v k+1 k px · qx+k
k=0 k=0
n−1
X
= vqx + v k+1 k px · qx+k
k=1
n−1
X
= vqx + vpx v k k−1 px+1 · qx+k
k=1
n−2
X
= vqx + vpx v j+1 j px+1 · qx+j+1 (using the substitution j = k − 1)
j=0
n−2
X
1
= vqx + vpx v j+1 j| qx+1 = vqx + vpx Ax+1:n−1
j=0

n| Ax = v n+1 n px · qx+n + n+1| Ax (3.24)


Proof:

X ∞
X
n| Ax = v k+1 k| qx = v k+1 k px · qx+k
k=n k=n

X
n+1
=v n px · qx+n + v k+1 k px · qx+k
k=n+1
n+1
=v n px · qx+n + n+1| Ax


Ax:n = vqx + vpx Ax+1:n−1 (3.25)


Proof:
n−1
X n−1
X
1
Ax:n = Ax:n + n Ex = v k+1 k| qx + v n n px = v k+1 k px · qx+k + v n n px
k=0 k=0
n−1
X
= vqx + v k+1 k px · qx+k + v n n px
k=1
n−1
!
X
k n−1
= vqx + vpx v k−1 px+1 · qx+k + v n−1 px+1
k=1
n−2
!
X
= vqx + vpx v j+1 j px+1 · qx+j+1 + v n−1 n−1 px+1 (using j = k − 1)
j=0

1

= vqx + vpx Ax+1:n−1 + n−1 Ex+1

= vqx + vpx Ax+1:n−1




Downloaded by Kipkoech Stanley (stanleykipkoech8@gmail.com)


lOMoARcPSD|3982611

CHAPTER 3. LIFE INSURANCE BENEFITS 37

There also exist recursive formulae for the second moments of the present value random variable,
however, since the proofs are similar to those for the first moment, we only prove the case for a whole
life insurance.

2
Ax = v 2 qx + v 2 px · 2 Ax+1 (3.26)
Proof:

X ∞
X
2 2(k+1)
Ax = v k| qx = v 2k+2 k px · qx+k
k=0 k=0

X
= v 2 qx + v 2k+2 k px · qx+k
k=1

X
= v 2 qx + v 2(j+1)+2 j+1 px · qx+j+1 (using j = k − 1)
j=0
X∞
= v 2 qx + v 2j+4 j px+1 · px · qx+j+1
j=0

2 2
X 2
= v qx + v p x v j+1 j px+1 · qx+j+1
j=0

= v 2 qx + v 2 px · Ax+1 2

For the other types of insurance the recursive formulae for the second moment are

2 1
Ax:n = v 2 qx + v 2 px · 2 Ax+1:n−1
1 2
Ax:n = v 2 qx + v 2 px · 2 Ax+1:n−1 2
n| Ax = v 2(n+1) n px · qx+n + 2n+1| Ax

Example: An individual aged 41 purchases a whole life insurance policy with benefit payable at the
end of the year of death. You are given

• i = 0.05

• p40 = 0.9972

• 2 A41 − 2 A40 = 0.00433

• A41 − A40 = 0.00822

Find the variance of the present value random variable.

Solution: We know that Var[Z] = 2 A41 − (A41 )2 so we need to find expressions for 2 A41 and A41 . Using
the recursive formula for a whole life insurance benefit,

A40 = vq40 + vp40 A41 = v(1 − p40 ) + vp40 A41


1 1
= (1 − 0.9972) + 0.9972A41
1.05 1.05

=⇒ A40 = 0.0026 + 0.949714A41

Downloaded by Kipkoech Stanley (stanleykipkoech8@gmail.com)


lOMoARcPSD|3982611

CHAPTER 3. LIFE INSURANCE BENEFITS 38

Since we are told that A41 − A40 = 0.00822, we have a set of simultaneous equations that may be solved
to give A41 = 0.215169.

Similarly,
2
A40 = v 2 qx + v 2 px · 2 A41 = v 2 (1 − px ) + v 2 px · 2 A41
 2  2
1 1
= (1 − 0.9972) + 0.9972 · 2 A41
1.05 1.05

2
=⇒ A40 = 0.002537 + 0.90452 A41
and since 2 A41 − 2 A40 = 0.00433, solving these equations gives 2 A41 = 0.071925. Substituting the values
found for A41 and 2 A41 into the formula for the variance gives

Var[Z] = 0.071925 − (0.215169)2 = 0.025627.

3.8 Benefit payments for mthly periods


Some insurance contracts specify that the benefit is to be paid at the end of a period shorter than a
year, for example, at the end of the month. Since the equations for the insurance functions are very
much the same as in the discrete case, we focus more on how we can approximate them using the UDD
assumption. When we considered the EPV of benefits paid at the end of the year of death we used Kx
as this represents the total number of years the life survives. For benefits paid at the end of the m1 th
(m)
of a year we introduce the random variable Kx which represents the future lifetime rounded to the
(m)
lower m1 th of a year. Using the floor function, Kx is defined as

1
Kx(m) = ⌊mTx ⌋
m
(2)
For example, suppose a life aged x survives for a further 23.675 years, then Kx = 23, Kx = 23.5,
(4) (12) 8
Kx = 23.5, and Kx = 23 12 .
(m)
The probability mass function for Kx is given by
   
(m)
 1 1
Pr Kx = k = Pr k < Tx < k + = Pr (Tx > k) − Pr Tx > k +
m m
= k px − k+ 1 px
m

= k| 1 qx
m

We now use this to give expressions for the insurance functions when a benefit is payable at the end of
the m1 th year of death, as before, the present value random variable is denoted by Z
(m) 1 (m) P∞ k+1
+m
Whole life insurance: Z = v Kx =⇒ E[Z] = Ax = k=0 v m · k 1
| qx
m m

( (m) 1
+m (m) 1
v Kx if Kx ≤ n − Pmn−1 k+1
Term insurance: Z= (m)
m =⇒ A(m)1x:n = k=0 v m k 1
| qx
0 if Kx ≥ n m m

( (m) 1
+m (m) 1
v Kx if Kx ≤ n − (m)
Endowment insurance: Z= (m)
m =⇒ Ax:n = A(m)1x:n + n Ex
vn if Kx ≥ n

Downloaded by Kipkoech Stanley (stanleykipkoech8@gmail.com)


lOMoARcPSD|3982611

CHAPTER 3. LIFE INSURANCE BENEFITS 39


(
(m) 1
0 if Kx ≤ n − m (m) P∞ k+1
Deferred insurance: Z= (m) 1 (m) =⇒ n| Ax = k=mn v m k 1
| qx
v Kx + m if Kx ≥ n m m

These equations are not always efficient, for example, if we had a 10 year term insurance with benefit
payable at the end of the month of death our summation would include 120 terms. We therefore
introduce a set of equations based on the UDD assumption that gives an mthly death benefit EPV in
terms of an annual death benefit EPV. A proof is given for the whole life case, this can be extended for
the remaining equations.

i
Ax(m) = Ax (3.27)
i(m)

Proof: We can consider a whole life insurance as the sum of deferred one-year term insurances. In the
first year we have A(m)1x:1 but in the second year the term insurance A(m) x+1:1
1
is contingent on the life
survivng the first year and must be discounted by one year. Therefore, the whole life insurance may be
written as

X
Ax(m) = v k k px A(m) x+k:1
1

k=0

where
m−1 m−1   m−1  
X j+1 X j+1 X j+1
A(m) x+k:1
1
= v m j 1
| qx+k = v m j px+k − j+1 px+k = v m j+1 qx+k − j qx+k
m m m m m m
j=0 j=0 k=0

j j+1
From the UDD assumption we know that for 0 < s ≤ 1, t qx = tqx and since 0 < m
< m
≤ 1.
Applying this to the previous line gives
m−1   m−1
X j+1 j+1 j qx+k X j+1
A(m) x+k:1
1
= v m qx+k − qx+k = vm
j=0
m m m j=0

Using standard results about geometric series we can evaluate the sum as follows
m−1
" 1
#
m
q x+k 1
X  1 j q
x+k 1 1 − (v m)
A(m) x+k:1
1
= vm vm = · vm 1
m j=0
m 1 − vm
 
qx+k 1−v iv
= · −1 = qx+k 1
m vm −1 m((1 + i) m − 1))
iv
= qx+k (m)
i
(m)
Substituting this expression into the initial equation for Ax gives
∞ ∞
X i i X i
Ax(m) = v k+1
k px qx+k (m) = v k+1 k| qx = Ax
k=0
i i(m) k=0
i(m)


The formulae for the other insurance functions are

i i (m) i
A(m)1x:n = (m)
1
Ax:n (m)
n| Ax = A
(m) n| x
Ax:n = 1
Ax:n + n Ex (3.28)
i i i(m)

Downloaded by Kipkoech Stanley (stanleykipkoech8@gmail.com)


lOMoARcPSD|3982611

CHAPTER 3. LIFE INSURANCE BENEFITS 40

1
where i(m) is the interest rate convertible m
thly and satisfies
m
i(m)

1+i= 1+
m

Example: Consider a 3 year term insurance with sum insured £1000 issued to a select life aged 50.
The benefit is payable at the end of the quarter of death, you may assume that deaths are uniformly
distributed and mortality follows the life table given below. Calculate the EPV of the benefit given the
interest rate is i = 5%.
[x] l[x] l[x]+1 lx+2 x+2
50 9706 9687 9661 52
51 9680 9660 9630 53
52 9653 9629 9596 54

Solution: We need to find 1000A(4) [50]:3


1
but since we are told the deaths are uniformly distributed, we
1
can instead find A[50]:3 and use equation (3.28).
2
X 2
X
1
A[50]:3 = v k+1 k| q[50] = v k+1 (k p[50] − k+1 p[50] )
k=0 k=0
2
X l[50]+k − l[50]+k+1
= v k+1 ·
k=0
l[50]
 2  3
1 9706 − 9687 1 9687 − 9661 1 9661 − 9630
= · + · + · = 0.0070523
1.05 9706 1.05 9706 1.05 9706
The interest rate convertible quarterly is
 1

i(4) = 4 (1.05) 4 − 1 = 0.0490889

and therefore,
i 0.05
A(4) [50]:3
1
= 1
A[50]:3 = (0.0070523) = 0.0071832
i(4) 0.0490889
Finally, the EPV of the £1000 benefit is

£1000A(4) [50]:3
1
= £1000 × 0.0071832 = £7.1832

3.9 Continuous and discrete relationships


In the previous lecture we found formulae that relate the EPV of annual and mthly death benefits,
we now derive relationships linking the EPV of annual and continuous benefits for the main types of
insurance. The formulae are again based on the UDD assumption and we only prove the whole life case
as this may be extended for the remaining types of insurance.

i
Āx = Ax (3.29)
δ

Downloaded by Kipkoech Stanley (stanleykipkoech8@gmail.com)


lOMoARcPSD|3982611

CHAPTER 3. LIFE INSURANCE BENEFITS 41

Proof: Starting from the defintion of Āx ,


Z ∞ Z ∞
−δt
Āx = e fx (t) dt = e−δt t px µx+t dt
0 0
∞ Z
X k+1
= e−δt t px µx+t dt
k=0 k
X∞ Z 1
= e−δ(s+k) s+k px µx+s+k ds (using the substitution s = t − k)
k=0 0

X∞ Z 1
−δk
= k px e e−δs s px+k µx+s+k ds
k=0 0
∞ 1

X Z  
k+1 δ(1−s)
= k px v e s px+k µx+s+k ds multiplying by δ = 1
k=0 0 e
X∞ Z 1
k+1
= v k px qx+k eδ(1−s) ds
k=0 0

Ax  −δ(s−1) 1 Ax  δ  Ax
=− e 0
= e − 1 = [(1 + i) − 1]
δ δ δ
i
= Ax
δ


To get from line 5 to 6 we recall equation (2.7) which states that under the UDD assumption fx (t) = qx ,
therefore, s px+k µx+s+k = fx+k (s) = qx+k . The corresponding equations for term, deferred, and
endowment insurance functions are

1 i 1 i i 1
Āx:n = · Ax:n n| Āx = · n| Ax Āx:n = · Ax:n + n Ex
δ δ δ

Example: For a fully continuous whole life insurance issued to a life aged 40 you are given

• The death benefit is £1000 in the first year and increases by £500 each year for the following 3
years before remaining level at £5000 thereafter.

• Mortality follows the Life Table at the end of the lecture note at i = 5%.

• Deaths are uniformly distributed between integer ages.

Calculate the EPV of this benefit. If we consider splitting the EPV as shown in the graph below

Downloaded by Kipkoech Stanley (stanleykipkoech8@gmail.com)


lOMoARcPSD|3982611

CHAPTER 3. LIFE INSURANCE BENEFITS 42

then it may be written as

EPV = 1000Ā40 + 500E40 Ā41 + 5002 E40 Ā42 + 5003 E40 Ā43 + 25004 E40 Ā44

Note that we could have also split the EPV into a series of term insurances, however, since we are using
the Illustrative Life Table, whole life insurances are more suitable.
Since the UDD assumption holds
i
EPV = 500 · [2A40 + E40 A41 + 2 E40 A42 + 3 E40 A43 + 54 E40 A44 ]
δ 
i l41 2 l42 3 l43 4 l44
= 500 · 2A40 + v A41 + v A42 + v A43 + 5v A44
δ l40 l40 l40 l40

Substituting the appropriate values from the Illustrative Life Table gives
     
500 × 0.05 1 99285.88 5 99104.33
EPV = 2(0.12106) + (0.12665) + ... + (0.14496)
ln(1.05) 1.05 99338.26 1.054 99338.26

= £613.4038

3.10 Variable insurance benefits


The last example, apart from demonstrating why the approximation of continuous time assurances by
discrete is useful, it presented us with an additional challenge, the benefit changed according to the time
of death (it was not level). We could easily deal with this feature of the problem since the changes in
the benefit where over a few years and then it would level again. In this section we consider situations
where the insurance benefit increases (we will see that it can also decrease in some cases) in a systematic
way depending on the time of death.

Continuous Case
We first consider an insurance policy with arithmetically increasing benefit Tx payable immediately on
death, that is, the benefit is equal to the number of years a life currently aged x survives for.
Since the benefit is equal to Tx , the present value random variable is

Z = Tx v Tx = Tx e−δTx

Downloaded by Kipkoech Stanley (stanleykipkoech8@gmail.com)


lOMoARcPSD|3982611

CHAPTER 3. LIFE INSURANCE BENEFITS 43

and the EPV is given in actuarial notation by


Z ∞ Z ∞
(I¯Ā)x = te −δt
fx (t) dt = te−δt t px µx+t dt (3.30)
0 0

The I here implies the benefit is increasing and the bar above the I denotes that the increases are
continuous. For an increasing term insurance the corresponding formula is
Z n Z n
(I¯Ā)x:n
1
= te −δt
fx (t) dt = te−δt t px µx+t dt . (3.31)
0 0

An interesting case is when the increase in the benefit is geometric. In this case the expected present
value for the term assurance becomes (there is not much scope for whole life policies as the EPV will
fairly quickly become too high to be realistic). In this case the benefit paid at the time of death is
(1 + j)t . The EPV for the term insurance is then:
Z n
EPV = (1 + j)t v t fx (t) dt
Z0 n
1+j t
= ( ) fx (t) dt
1+i
Z0 n
= v ∗t fx (t) dt
0

1
= (Ā)x:n ,
i∗

1+i
where i∗ = 1+j
− 1.

Discrete Case
Now consider a discrete increasing benefit that increases by 1 each year, a benefit of 1 is paid at the
end of the year of death given death occurs in the first year, 2 in the second year, and k + 1 if death
occurs between the ages of x + k and x + k + 1. The present value random variable is therefore defined
as
(Kx + 1)v Kx +1

The EPV of the death benefit is then denoted in actuarial notation by (IA)x where


X
(IA)x = v k+1 (k + 1)k| qx (3.32)
k=0

The corresponding function for term assurance is


n
X
1
(IA)x:n = v k+1 (k + 1)k| qx . (3.33)
k=0

A useful relationship between term and whole increasing assurances is the following (proof is an exercise):

1
(IA)x:n = (IA)x − v n n px [nAx+n + (IA)x+n ] (3.34)

Downloaded by Kipkoech Stanley (stanleykipkoech8@gmail.com)


lOMoARcPSD|3982611

CHAPTER 3. LIFE INSURANCE BENEFITS 44

Remark: Equations for the EPV can be derived for a number of other patterns including benefits
that decrease over time, though for the latter only term assurances can be considered in order to avoid
the possibility of negative benefits.

Remark: One can consider benefit increases that take place more often than annually in a similar
way.

Example: Consider a 20-year increasing term assurance. The benefit is payable at the end of year
of death and it is £10, 000 in the first year increasing by £100 in every subsequent year. Assume i = 5%
and mortality that follows the tables at the end of the lecture notes. Find the EPV of this assurance.

1 1
EPV = (10000 − 100)A50:20 + 100IA50:20
l70
= 9900(A50 − v 20 A70 )
l50
l70
+ 100 (IA)50 − v 20 [20A70 + (IA)70 ]
 
l50
1 91082.43
= 9900(0.18931 − 0.42818)
1.0520 98576.37
 1 91082.43 
+ 100 5.825499653 − [20 · 0.42818 + 6.742450463]
1.0520 98576.37
= 1923.705

Example: Consider a 20-year decreasing term assurance. The benefit is payable at the end of year
of death and it is £10, 000 in the first year decreasing by £100 in every subsequent year. Assume i = 5%
and mortality that follows the tables at the end of the lecture notes. Find the EPV of this assurance.

1 1
EPV = (10000 + 100)A50:20 − 100IA50:20
l70
= 10100(A50 − v 20 A70 )
l50
l70
− 100 (IA)50 − v 20 [20A70 + (IA)70 ]
 
l50
1 91082.43
= 10100(0.18931 − 0.42818)
1.0520 98576.37
 1 91082.43 
− 100 5.825499653 − 20
[20 · 0.42818 + 6.742450463]
1.05 98576.37
= 11862.495.

Downloaded by Kipkoech Stanley (stanleykipkoech8@gmail.com)


lOMoARcPSD|3982611

Chapter 4

Life Annuities

4.1 Introduction to life annuities


In this chapter we derive expressions for the expected present value of annuities contingent on the
survival of an individual and show how they may be related to the insurance functions discussed in
the previous chapter. Life annuities will also be a critical part of premium calculation as shown in the
following chapter. We will use results and notation from the valuation of annuities certain so we begin
with a review of these.

An n-year annuity due is the present value of a series of unit payments made at the beginning of each
year from t = 0 to t = n − 1, we have the equation
n−1
X 1 − vn
än = vk = (4.1)
k=0
d

An n-year annuity immediate is the present value of a series of unit payments made at the end of each
year, although it is often easier to think of this as the beginning of the following year. The payments
are therefore made at t = 1 through to t = n and we have
n
X 1 − vn
an = vk = (4.2)
k=1
i

An n-year continuous annuity is the present value of continual payments such that one unit is paid
across each year, the formula is given by
Z n
1 − e−δn
ān = e−δt dt = (4.3)
0 δ

1
For both annuities due and immediate we can consider the case where a unit payment is made every m
years in installments of m1 , the formulae are
mn−1 m
1 − vn d(m)

(m) 1 X k
än = v m = (m) where 1 − d = 1− (4.4)
m k=0 d m

mn m
1 − vn i(m)

(m) 1 X k
an = v m = (m) where 1 + i = 1+ (4.5)
m k=1 i m

Since premiums tend to start immediately on issue we focus more on annuities due and continuous
annuities.

Downloaded by Kipkoech Stanley45


(stanleykipkoech8@gmail.com)
lOMoARcPSD|3982611

CHAPTER 4. LIFE ANNUITIES 46

Whole-life annuity due


Consider an annuity of 1 per year payable annually in advance given the individual is still alive, the first
payment occurs at t = 0 when the policy is issued and the final payment will occur at the beginning
of the year of death when t = Kx . There are Kx + 1 payments in total so the present value random
variable is therefore given by
Y = äKx +1
The EPV of a whole life annuity due is then given in actuarial notation by

X ∞
X
äx = E[Y ] = äk+1 Pr(Kx = k) = äk+1 · k| qx
k=0 k=0

Since an annuity certain is not easy to evaluate when inside a sum, we can use equation (4.1) to find a
simpler formula for the EPV.
∞ ∞  ∞
1 − v k+1

X X 1X
1 − v k+1 k px − 1 − v k+1 k+1 px
 
äx = äk+1 · k| qx = (k px − k+1 px ) =
k=0 k=0
d d k=0
1
(1 − v) + (1 − v 2 )px + (1 − v 3 )2 px + ... − (1 − v)px + (1 − v 2 )2 px + (1 − v 3 )3 px + ...
 
=
d
1
(1 − v) + (v − v 2 )px + (v 2 − v 3 )2 px + ...

=
d
1
(1 − v) + v(1 − v)px + v 2 (1 − v)2 px + ...

=
d
1−v 
1 + vpx + v 2 2 px + ...

=
d
X∞
= v k k px
k=0

Therefore, the EPV of a whole life annuity due is



X
äx = v k k px (4.6)
k=0

We may also take a less formal approach to deriving equation (4.6) by representing the cash flow using
a timeline. By considering the amount paid at the start of each year, the appropriate discount factor,
and the probability that each payment occurs, we can find the EPV of each individual payment.

The total EPV is then the sum of the EPV for all individual payments, therefore, the EPV of a whole
life annuity due is

X
2 3
äx = 1 + vpx + v 2 px + v 3 px + ... = v k k px
k=0

Downloaded by Kipkoech Stanley (stanleykipkoech8@gmail.com)


lOMoARcPSD|3982611

CHAPTER 4. LIFE ANNUITIES 47

Although equation (4.6) is a useful equation for äx , it may not be used to find expressions for higher
moments or the variance of Y . This is because the individual terms are dependent, the probability that
the life survives two years depends on whether they survived the first year. If we needed to calculate
the second moment of Y we would need to use the original expression involving an annuity certain, that
is,

X 2
E[Y 2 ] = äk+1 k| qx
k=0

As before, this is not an easy approach to take so we now show how the expectation and variance may
be calculated using the whole-life insurance functions defined in the previous chapter. Recall that the
present value random variable for a whole life insurance benefit payable at the end of the year of death
is Z = v Kx +1 , using equation (4.1)
1 − v Kx +1
Y = äKx +1 =
d
so the expectation of Y is

1 − E v Kx +1
 
1 − v Kx +1
 
1 − Ax
E[Y ] = E = =
d d d

We therefore have the relationship

1 − Ax
äx = =⇒ Ax = 1 − däx (4.7)
d

Now recall that the variance of the whole life present value random variable is Var[Z] = 2 Ax − (Ax )2 ,
since the variance of a constant is zero, we have

1 − v Kx +1
 
1 1
= 2 Var v Kx +1 = 2 2 Ax − (Ax )2
  
Var[Y ] = Var
d d d

4.2 Temporary and deferred life annuities


Temporary life annuity
A temporary life annuity due pays a unit benefit at the beginning of each year, provided the individual
is alive, for a maximum of n years. If the life dies within the n-year term they will receive payments
starting from t = 0 through to the beginning of the year of death, a total of Kx + 1 payments. If the
life survives the whole duration of the term they will recieve the full n payments. The present value
random variable is given by

äKx +1 if Kx ≤ n − 1
Y = = ämin(Kx +1,n)
än if Kx ≥ n

For this life annuity it is much easier to find the EPV using the less formal approach and consider the
cash flow as a time-line.

Downloaded by Kipkoech Stanley (stanleykipkoech8@gmail.com)


lOMoARcPSD|3982611

CHAPTER 4. LIFE ANNUITIES 48

If we consider the EPV for each individual payment separately we see that the total EPV may be
written as

E[Y ] = äx:n = 1 + vpx + v 2 2 px + ... + v n−1 n−1 px

Therefore, our formula to calculate the EPV of a temporary life annuity due is

n−1
X
äx:n = v k k px (4.8)
k=0

As with the whole life annuity we can derive expressions relating the expectation and variance of Y
with the insurance functions defined previously. Recall that the present value random variable for an
endowment insurance benefit is given by Z = v min(Kx +1,n) , then

1 − E v min(Kx +1,n)
 
1 − v min(Kx +1,n)
 
h i 1 − Ax:n
E[Y ] = E ämin(Kx +1,n) = E = =
d d d

Hence,
1 − Ax:n
äx:n = =⇒ Ax:n = 1 − däx:n (4.9)
d
1
It is important to note that although equation (4.8) may resemble the equation for Ax:n , the EPV of
a term insurance benefit, the annuity is in fact related to the endowment insurance function Ax:n . An
expression for the variance of Y may be found in a similar way

1 − v min(Kx +1,n)
 
1 1
= 2 Var v min(Kx +1,n) = 2 2
Ax:n − (Ax:n )2
  
Var[Y ] = Var
d d d

Deferred life annuity


A deferred annuity is an annuity where the payments begin at some specified future time. Consider
the case where an individual currently aged x purchases an insurance policy where premiums are paid
according to a deferred annuity due with n-year deferred period. The annuity payments will be made
at t = x + n and at the beginning of each subsequent year provided the individual is alive, therefore,
once the deferred period has ended the annuity is equivalent to a whole life annuity. The present value
random variable is therefore the same as for a whole life annuity with the initial n-years subtracted.
This will, however, give äKx +1−n which represents the value at t = x + n, to get the value at t = 0 we
must discount the annuity by n years. Taking this into account, the present value random variable may
be written as

0 if Kx ≤ n − 1
Y = n
v äKx +1−n if Kx ≥ n

A timeline showing the cash flow for a deferred annuity due is given on the following page, using this
approach the EPV of a deferred annuity due is

n| äx = v n n px + v n+1 n+1 px + v n+2 n+2 px + ...


X
=⇒ n| äx = v k k px (4.10)
k=n

Downloaded by Kipkoech Stanley (stanleykipkoech8@gmail.com)


lOMoARcPSD|3982611

CHAPTER 4. LIFE ANNUITIES 49

The expression for the second moment of Y is too complex and since Y can not easily be written in
terms of the present value random variable for any insurance function we do not consider an equation
for the variance. However, if asked to find the variance we can do so as we would for any discrete
random variable.

Example: A life currently aged 95 purchases an insurance contract and pays premiums at the start of
each year starting in two years time. Mortality follows the life table given below and the interest rate
is 6%. Find the EPV of the annuity and the variance of the present value random variable.
x 95 96 97 98 99 100
lx 1000 750 400 225 75 0

Solution: Using equation (4.10) the EPV of a 2-year deferred annuity due is
∞ 4  
X
k
X
k l95+k
2| ä95 = v k p95 = v
k=2 k=2
l95

l97 l98 l99


= v2 + v3 + v4
l95 l95 l95

1 400 1 225 1 75
= 2
· + 3
· + 4
·
1.06 1000 1.06 1000 1.06 1000

= 0.6043

In this case the present value random variable is given by

1 − v Kx −1

0 if Kx ≤ 1
Y = where ä =
v 2 äKx +1−2 if Kx ≥ 2 Kx −1
d

and the variance may be calculated by evaluating the first and second moments using the following
table.
k Pr(Kx = k) = k| qx Y Y Pr(Kx = k) Y 2 Pr(Kx = k)
0 0.25 0 0 0
1 0.35 0 0 0
2 0.175 0.8900 0.1557 0.1386
3 0.15 1.7296 0.2594 0.4487
4 0.075 2.5217 0.1891 0.4769
5 0 3.6730 0 0
5
X 5
X
E[Y ] = Y Pr(Kx = k) = 0.60432 , E[Y 2 ] = Y 2 Pr(Kx = k) = 1.06428
k=0 k=0
2
=⇒ Var[Y ] = E[Y 2 ] − (E[Y ]) = 1.06428 − (0.60432)2 = 0.699075

Downloaded by Kipkoech Stanley (stanleykipkoech8@gmail.com)


lOMoARcPSD|3982611

CHAPTER 4. LIFE ANNUITIES 50

Recall that the deferred insurance function may be written as a discounted whole life insurance function,
the same is also true for annuities, that is

n| äx = n Ex äx+n (4.11)

Proof:

X ∞
X
k
n| äx = v k px = v j+n j+n px (using j = k − n)
k=n j=0

X
n
= v n px v j j px+n
j=0

= n Ex äx+n


We can also show that the sum of a deferred and temporary annuity is equal to a whole life annuity

äx = äx:n + n| äx (4.12)


Proof:
n−1
X ∞
X ∞
X
k k
äx:n + n| äx = v k px + v k px = v k k px
k=0 k=n k=0


Finally, as with the insurance functions, we can derive a recursive formula for a whole life annuity due
which could be used to create a table of the EPV for consecutive ages.

äx = 1 + Ex äx+1 (4.13)


Proof:

X ∞
X
k
äx = v k px = 1 + v k k px
k=0 k=1

X
=1+ v j+1 j+1 px (using j = k − 1)
j=0

X
= 1 + vpx v j j px+1
j=0

= 1 + Ex äx+1


4.3 Guaranteed annuities


It is often the case in pension benefits that the annuity is guaranteed to be paid for some period
regardless of whether or not the life dies before the end of the period. An n-year guaranteed annuity
due pays 1 at the start of every year for the first n years and continues to pay afterwards if the life is
still alive. Since the first n payments are not contingent on the survival of the life, the present value
is given by an n year annuity certain än . If the life survives for more than n years the payments are
made until the beginning of the year of death so the present value is äKx +1 .

Downloaded by Kipkoech Stanley (stanleykipkoech8@gmail.com)


lOMoARcPSD|3982611

CHAPTER 4. LIFE ANNUITIES 51

Therefore, by combining both cases the present value random variable is



än if Kx ≤ n − 1
Y =
äKx +1 if Kx ≥ n

By manipulating the expression for Y we can derive an equation for the EPV of a guaranteed annuity
 
än if Kx ≤ n − 1 än if Kx ≤ n − 1
Y = =
äKx +1 if Kx ≥ n än + äKx +1 − än if Kx ≥ n


0 if Kx ≤ n − 1
= än +
äKx +1−n if Kx ≥ n

= än + Y ∗

where Y ∗ is the present value random variable for a deferred annuity due. The expected value of Y is
denoted in actuarial notation by äx:n and is therefore

E[Y ] = E[än + Y ∗ ] = än + n| äx =⇒ äx:n = än + n| äx (4.14)

By using equation (4.11) we arrive at the more commonly used expression

äx:n = än + n Ex äx+n (4.15)

Example: You are given the following information


• ä50:10 = 17.5661

• ä50:20 = 18.0606

• ä60 = 14.134

• ä70 = 10.375

• i = 4%
Find ä60:10 .

Solution: We know from equation (4.12) that a temporary annuity may be written as a whole life
annuity minus a deferred annuity

=⇒ ä60:10 = ä60 − 10| ä60 = ä60 − 10 E60 ä70

We know the values of ä60 and ä70 so we only need to determine 10 E60 . Using equation (4.15) and the
first bullet point, we have

1 − v 10
ä50:10 = ä10 + 10 E50 ä60 = + 10 E50 ä60
d
1
10
1 − 1.04
=⇒ 17.5661 = 1
 + 14.134 · 10 E50
1 − 1.04

⇐⇒ 10 E50 = 0.6460

Downloaded by Kipkoech Stanley (stanleykipkoech8@gmail.com)


lOMoARcPSD|3982611

CHAPTER 4. LIFE ANNUITIES 52

Similarly, for the second bullet point

1 − v 20
ä50:20 = ä20 + 20 E50 ä70 = + 20 E50 ä70
d
1
20
1 − 1.04
=⇒ 18.0606 = 1
 + 10.375 · 20 E50
1 − 1.04

⇐⇒ 20 E50 = 0.3785

Pure endowment insurance functions satisfy a similar multiplicative property as survival functions, so

10 E50 · 10 E60 = v 10 10 p50 · v 10 10 p60 = v 20 20 p50 = 20 E50

20 E50 0.3785
⇐⇒ 10 E60 = = = 0.5859
10 E50 0.6460

We now have the three values necessary to evaluate the temporary annuity, therefore

ä60:10 = ä60 − 10 E60 ä70 = 14.134 − (0.5859 × 10.375) = 8.0553

Life annuities payable m times per year


In practice premiums are not generally paid every year but instead at more frequent intervals such as
every month. Here we give the present value random variable and EPV for a whole-life, temporary, and
deferred annuity due with payments m times a year.

Whole-life annuities payable m times a year


Consider a life aged x who pays premiums at the start of each month until death with the first premium
paid immediately. The final payment will be made at the beginning of the m1 th year of death and recall
(m)
that we represent this by the random variable Kx . The life will therefore make Kx + m1 payments in
total so for a whole life annuity due with m payments, the present value random variable is
(m) 1
+m
1 − v Kx
Y = ä (m) 1
=
Kx +m d(m)
(m) 1
+m
If we then notice that v Kx is the present value random variable for a whole life insurance benefit
1
payable at the end of the m
th year of death, the EPV of the annuity may be expressed in actuarial
notation as
(m)
1 − Ax
äx(m) = (4.16)
d(m)

Alternatively, using the timeline given on the following page and considering the EPV of each individual
payment
1  1 2 3

äx(m) = 1 + v m 1 px + v m 2 px + v m 3 px + ...
m m m m


1 X k
=⇒ äx(m) = v m k px (4.17)
m k=0 m

Downloaded by Kipkoech Stanley (stanleykipkoech8@gmail.com)


lOMoARcPSD|3982611

CHAPTER 4. LIFE ANNUITIES 53

The variance of Y is given by


(m)
" 1
#
+m
1 − v Kx 1 2
Ax(m) − (Ax(m) )2 .

Var[Y ] = Var = 2
d(m) (d(m) )

Temporary annuities payable m times a year


The present value random variable is given by
(m) 1
(m) 1 − v min(Kx +m ,n)
Y = ä =
min(Kx
(m) 1
+m ,n) d(m)
(m) 1
and since v min(Kx + m ,n) is the present value random variable for an n-year endowment insurance benefit
payable at the end of the m1 th period, the EPV of the annuity is given in actuarial notation by

(m)
(m) 1 − Ax:n
äx:n = (4.18)
d(m)

If we consider the timeline approach the formula for the temporary annuity differs from the whole life
annuity only in the limit of the sum. For the temporary annuity we have a total of mn payments and
since the first payment is made immediately we sum from k = 0 to k = mn − 1. Therefore,

mn−1
(m) 1 X k
äx:n = v m k px (4.19)
m k=0 m

The variance of Y is
(m)
" 1
#
1 − v min(Kx +m ,n)
1 
2 (m) (m)

Var[Y ] = Var = 2 Ax:n − (Ax:n )2
d(m) (d(m) )

Deferred annuities payable m times per year


The present value random variable for an n-year deferred annuity payable m times a year is

(m)
 0 if Kx ≤ n − m1
Y = (m) (m)
if Kx ≥ n
 ä (m) 1
Kx + m −n

As with the annual case, it is much easier to determine the EPV using the timeline approach, doing so
gives

(m) 1 X k
n| äx = v m k px (4.20)
m k=mn m

We do not consider an expression for the variance of Y for a deferred annuity payable m times a year.

Downloaded by Kipkoech Stanley (stanleykipkoech8@gmail.com)


lOMoARcPSD|3982611

CHAPTER 4. LIFE ANNUITIES 54

4.4 The UDD assumption for life annuities


The formulae given in the previous lecture for annuities payable m times a year are not useful when
considering longer contracts or large m due to the number of terms that occur in each sum. For this
reason we give the following equation based on the UDD assumption that relates an annuity with annual
payments and an annuity payable m times a year.

id i − i(m)
äx(m) = α(m)äx − β(m) where α(m) = , β(m) = (4.21)
i d(m)
(m) i(m) d(m)
(m)
Proof: Recall that under the UDD assumption we have the following equation relating Ax and Ax
i
Ax(m) = Ax
i(m)
We may then write a whole life annuity due payable m times a year as
(m) i

(m) 1 − Ax 1 − i(m) Ax
äx = (m)
= (m)
=⇒ i(m) d(m) ä(m)
x = i(m) − iAx
d d

=⇒ i(m) d(m) ä(m)


x = i(m) − i(1 − däx ) (using equation (4.7))

i(m) − i(1 − däx ) i(m) − i + idäx idäx i − i(m)


⇐⇒ äx(m) = = = −
i(m) d(m) i(m) d(m) i(m) d(m) i(m) d(m)

= α(m)äx − β(m)


Example: (Q3 Jan. 2013) A 30-year temporary life annuity due is issued to a life aged 45 with
benefits payable monthly. For the first 10 years the benefit is £10, increasing to £30 thereafter.
Mortality follows the Tables at the end of the lecture notes with i = 5%. Calculate the EPV of
this annuity.

Solution: Since the benefit changes during the term of the contract we need to consider the EPV in
two parts. We can think of the first 10 years as a temporary annuity paying £10 a month and the
second 20 years also as a temporary annuity but instead paying £30 a month and starting from age
55. Note that we must multiply the second annuity by 10 E45 to ensure we find the present value and
consider the probability that the life survives to age 55. Remember also that a unit annuity due payable
m times a year pays m1 at the start of each period, not 1, so when evaluating the EPV we need to make
sure we use the total amount paid each year. Taking all of this into account the EPV may be written
as
(12) (12)
EPV = 120ä45:10 + 36010 E45 ä55:20

   
(12) (12) (12) (12)
= 120 ä45 − 10 E45 ä55 + 36010 E45 ä55 − 20 E55 ä75 (using equations (4.11) and (4.12))

(12) (12) (12)


We can now evaluate ä45 , ä55 and ä75 using equation (4.21).
(12)
ä45 = α(12)ä45 − β(12) = (1.000197 × 17.81621) − 0.466508 = 17.35321
(12)
ä55 = α(12)ä55 − β(12) = (1.000197 × 16.05987) − 0.466508 = 15.59653

Downloaded by Kipkoech Stanley (stanleykipkoech8@gmail.com)


lOMoARcPSD|3982611

CHAPTER 4. LIFE ANNUITIES 55

(12)
ä75 = α(12)ä75 − β(12) = (1.000197 × 10.31778) − 0.466508 = 9.853305

Since 10 E45 = 0.6065504 and 20 E55 = 0.3281915, the total EPV is

EPV = 120 × (17.35321 − (0.6065504 × 15.59653))


+ 360 × 0.6065504 × (15.59653 − (0.3281915 × 9.853305))
= £3646.685

Life annuities immediate


Since premiums are more commonly paid at the start of a period we have so far only considered annuities
due, however, we now use the principles already discussed and apply them to annuities immediate.

Whole life annuity immediate


For a whole life annuity immediate the initial payment is made at t = 1 instead of t = 0 as it is for an
annuity due, however, for both annuities the final payment will be made at the beginning of the year
of death. Therefore, for an annuity immediate there is a total of Kx payments and so the present value
random variable is
Y = aKx
The expected value of Y is then

1 − v1 E v Kx +1
 
1 − v Kx
 
  1 − (1 + i)Ax
a x = E aK x =E = =
i i i

and the variance is


1 − (1 + i)v Kx +1
   Kx +1 
v 1 2
Ax − (Ax )2
  
Var aKx = Var = Var = 2
i iv d

An alternative formula for the EPV of a whole life annuity immediate may be found by considering the
sum of the EPV of each individual payment, this gives

X
ax = vpx + v 2 2 px + v 3 3 px + ... = v k k px (4.22)
k=1

This equation can now be used to derive an expression relating the EPV of a whole life annuity due
and annuity immediate.

X ∞
X
k
äx = v k px = 1 + v k k px = ax + 1 =⇒ äx = ax + 1 (4.23)
k=0 k=1

Temporary annuity immediate


For a temporary annuity immediate unit payments are made at t=1,2,...,n, conditional on the survival
of the individual. The present value random variable is

Y = amin(Kx ,n)

and by summing the EPV of the individual payments, we have


n
X
2 n
ax:n = vpx + v 2 px + ... + v n px = v k k px (4.24)
k=1

Downloaded by Kipkoech Stanley (stanleykipkoech8@gmail.com)


lOMoARcPSD|3982611

CHAPTER 4. LIFE ANNUITIES 56

If we compare this to a temporary annuity due we can see that the annuity due has an additional
payment at t = 0 whilst the annuity immediate has an additional payment at t = n, therefore, we have
the relationship
1 + ax:n = äx:n + v n n px (4.25)

4.5 Life annuities payable continuously


Whole life annuity payable continuously
In practice annuities are payable at discrete time intervals, however, should these intervals be small,
for example weekly, it is often more convenient to treat the payments as being made continuously. A
whole life annuity payable continuously pays a rate of 1 per year provided the individual is alive. Since
the annuity pays up until the moment of death the present value random variable is given by

Y = āTx

If we recall that the present value random variable for a whole life insurance benefit payable at the time
of death is Z = e−δTx , then the expected value of Y may be written as

1 − E e−δTx
 
1 − e−δTx
 
E[Y ] = E =
δ δ

Therefore, in actuarial notation the EPV of a whole life annuity payable continuously is

1 − Āx
āx = (4.26)
δ

The variance of Y is given by

1 − e−δTx


1 1
= 2 Var e−δTx = 2 2
Āx − (Ax )2
  
Var[Y ] = Var
δ δ δ

An integral formula may also be derived by considering the expectation of Y as we usually would for a
continuous random variable
Z ∞ Z ∞
E[Y ] = āt fx (t) dt = āt · t px µx+t dt
0 0

We can evaluate this integral using integration by parts and noting that

d 1 − e−δt
 
d
ā = = e−δt
dt t dt δ

Therefore, ∞ Z ∞

1 − e−δt
Z 
d
āx = āt (−t px ) dt = −t px + e−δt t px dt
0 dt δ 0 0
Z ∞
=⇒ āx = e−δt t px dt (4.27)
0

Example: You are given that for a particular survival model µx+t = 0.03 for all x, t > 0. If the force
of interest is δ = 5%, calulate  
p
Pr Y ≥ E[Y ] − Var[Y ]

Downloaded by Kipkoech Stanley (stanleykipkoech8@gmail.com)


lOMoARcPSD|3982611

CHAPTER 4. LIFE ANNUITIES 57

where Y is the present value random variable for a whole life annuity payable continuously.
Solution: We first need to evaluate āx , Āx and 2 Āx so we can obtain values for E[Y ] and Var[Y ].
Z ∞ Z ∞ Z ∞
−δt −δt −µt
Āx = e t px µx+t dt = µe e dt = µ e−(µ+δ)t dt
0 0 0
µ  −(µ+δ)t ∞ µ
=− e 0
=
µ+δ µ+δ
0.03
= = 0.375
0.03 + 0.05
The rule of moments may be used to find 2 Āx by evaluating Āx at 2δ.

2 µ 0.03 3
=⇒ Āx = = =
µ + 2δ 0.03 + 0.1 13

Using equation (4.26) to evaluate āx gives

1 − Āx 1 − 0.375
āx = = = 12.5
δ 0.05
Therefore
 
1 2 2 1 3
− 0.3752

E[Y ] = āx = 12.5 and Var[Y ] = 2 Āx − (Āx ) = = 35.05769
δ 0.052 13

If we substitute these values into the probability the problem reduces to calculating

Pr(Y ≥ 12.5 − 35.05769) = Pr(Y ≥ 6.49519)

We know that the present value random variable Y is dependent on Tx so we can rearrange the
probability such that it is of the form Pr(Tx ≥ a) for some constant a. This is then just the definition
of the survival function which is already known.

1 − e−δTx
 
≥ 6.49519 = Pr e−δTx ≤ 1 − 6.49519δ

Pr(Y ≥ 6.49519) = Pr
δ
= Pr(−δTx ≤ ln(1 − 6.49519δ))
 
−ln(1 − 6.49519δ)
= Pr Tx ≥
δ
= Pr(Tx ≥ 7.8537)
= 7.8537 px = e−(0.03×7.8537) = 0.79

Hence  p 
Pr Y ≥ E[Y ] − Var[Y ] = 0.79

Temporary annuity payable continuously


An n-year temporary annuity payable continuously pays a rate of 1 per year for the next n years
provided the individual is alive. The present value random variable is given by

1 − e−δmin(Tx ,n)
Y = āmin(Tx ,n) =
δ

Downloaded by Kipkoech Stanley (stanleykipkoech8@gmail.com)


lOMoARcPSD|3982611

CHAPTER 4. LIFE ANNUITIES 58

and expected value of Y is

1 − E e−δmin(Tx ,n)
 
1 − e−δmin(Tx ,n)
 
E[Y ] = E =
δ δ

Using our knowledge of continuous time endowment insurances, the EPV is therefore

1 − Āx:n
āx:n = (4.28)
δ

and the variance is


1 2 2

Var[Y ] = Āx:n − ( Āx:n )
δ2
The integral expression more commonly used to evaluate the EPV of the annuity is similar to that for
the whole life case with the upper limit changed from infinity to n.
Z n
āx:n = e−δt t px , dt (4.29)
0


0.05 0 ≤ t ≤ 10
Example: You are given that µx+t = 0.02 for all x, t > 0 and δ =
0.10 t > 10
Calculate ā40 .

Solution: We must first note that since the force of interest changes during the term of the contract
we need to consider the first 10 years separately as a temporary annuity, that is,

ā40 = ā40:10 + 10 E40 ā50

Equations (4.27) and (4.29) can then be used to evaluate the annuities as follows, remember also that
for a constant force of mortality µ, the survival function for a life aged x is given by t px = e−µt
Z 10 Z ∞
−δt −10δ
ā40 = e t p40 dt + e 10 p40 e−δt t p50 dt
Z0 10 0
Z ∞
−0.05t −0.02t −0.5 −0.2
= e e dt + e e e−0.1t e−0.02t dt
Z0 10 Z ∞ 0

= e−0.07t dt + e−0.7 e−0.12t dt


0 0
1  −0.07t 10 e−0.7  −0.12t ∞
=− e 0
− e 0
0.07 0.12
−0.7
1   e
= 1 − e−0.7 +
0.07 0.12

= 11.3298

Note that the force of interest we use for 10 E40 is the the force of interest for the first 10 years as this
is the period we are discounting through.

4.6 Increasing Annuities


As with assurances, one can also consider annuities where the amount paid in each year changes from
year to year. There are many patterns one can consider, but we limit the discussion into simple
arithmetically increasing annuities.

Downloaded by Kipkoech Stanley (stanleykipkoech8@gmail.com)


lOMoARcPSD|3982611

CHAPTER 4. LIFE ANNUITIES 59

Whole life
Consider a whole life annuity due where the first payment is equal to £1 and every year the amount
increases by £1 so that in the second year it is £2, £3 in the third year and so on.
The expected present value of such an annuity is given by the current payments formula:

X
(Iä)x = v k (k + 1)k px . (4.30)
k=0

Values of (Iä)x and (Iä)[x] are also tabulated.


Example: Suppose now that we want to calculate the EPV for an increasing annuity immediate (defined
in complete analogy with the annuity due) for a life currently aged 40. We will use the tabulated values
for (Iä)x .
Solution: We can write down using the current payments formula:

(Ia)40 = vp40 + 2v 2 2 p40 + 3v 3 3 p40 + · · ·

and
(Iä)40 = 1 + 2vp40 + 3v 2 2 p40 + 4v 3 3 p40 + · · ·
Subtracting the two equations we get:

(Iä)40 − (Ia)40 = 1 + vp40 + v 2 2 p40 + v 3 3 p40 + · · · = ä40 .

So
(Ia)40 = (Iä)40 − ä40 = 288.17248 − 18.45776 = 269.71472.
From the above example it is clear that the following relationship holds for any x > 0.

(Iä)x − (Ia)x = äx . (4.31)

Temporary
Similarly we have
n−1
X
(Iä)x:n = v k (k + 1)k px . (4.32)
k=0

The values of temporary increasing life annuity due are not usually tabulated but one can use the
following property in order to express the temporary into whole life annuities due and whole life
increasing annuity due the values of which are tabulated.

lx+n
(Iä)x:n = (Iä)x − v n [näx+n + (Iä)x+n ]. (4.33)
lx

Note that when considering temporary life annuities we can also consider decreasing ones. That is
something we could not define for whole life annuities due to the risk of having negative payments. The
Temporary decreasing life annuity due is an annuity due that pays £n in the first year decreasing
by £1 every year thereafter. We can calculate its EPV using the following decomposition:

EPV = (n + 1)äx:n − (Iä)x:n . (4.34)

Example: Consider a 10−year life annuity due issued to a select life aged exactly x = [40]. The

Downloaded by Kipkoech Stanley (stanleykipkoech8@gmail.com)


lOMoARcPSD|3982611

CHAPTER 4. LIFE ANNUITIES 60

first payment is £50, 000 decreasing by £5, 000 per year thereafter. Calculate the EPV of this annuity
assuming mortality follows the tables at the back of the lecture notes and i = 5%.
Solution:
EPV = 55, 000ä[40]:10 − 5, 000(Iä)x:n
l50
= 55, 000(ä[40] − v 10 ä[40] )
l[40]
l50
− 5, 000((Iä)[40] − v 10 [10ä50 + (Iä)50 ].
l[40]

Using the values from the tables we calculate


l50 1 98, 576.37
v 10 = = 0.609268783
l[40] 1.0510 99, 327.82

and
EPV = 55, 000(18.45956 − 0.609268783 · 17.02453)
− 5, 000(288.2026 − 0.609268783[10 · 17.02453 + 235.17974]
= 238, 838.6.

We close the discussion of increasing annuities with the note that there are many possibilities to define
different kinds of increasing annuities. Indicatively, we mention the case of continuous annuity that
increases continuously:
Z n
¯
(Iā)x:n = te−δt t px dt
0
or a continuous annuity that the annual amount paid increases by £1 at the end of each year:
n−1
X
(Iā)x:n = (k + 1)k| āx:1 .
k=0

In any case the approach would be to try and write down the EPV using well known EPVs of
annuities.

Downloaded by Kipkoech Stanley (stanleykipkoech8@gmail.com)


lOMoARcPSD|3982611

Chapter 5

Premium Calculation and Policy Values

5.1 The Equivalence Principle


In this chapter we use our knowledge of insurance functions and annuities to develop a method of
calculating premiums known as the equivalence principle, to do so we introduce a new random variable
called the present value of future loss random variable.

An insurance policy is funded by contract premiums, this may be one single premium paid on issue or a
series of payments contingent on the survival of the policyholder. The premium will need to cover any
future benefits paid to the policyholder but also the expenses the insurance company incurs by creating
and managing the policy. There are two types of premium

Net premium: This premium only covers the benefit paid to the individual by the insurance company,
it does not allow for the insurance company’s expenses.

Gross premium: This premium covers both the benefit and the company’s expenses. There are three
main types of expense associated to a contract - initial expenses, renewal expenses and termination
expenses. These may include commission paid to an agent for selling a contract, staff salaries, and the
rent for the insurer’s premises.

Premiums are (usually) paid in advance, with the first premium payable when the policy is purchased.
For example, regular premiums for a policy on a single life cease to be payable on the death of the PH.
Sometimes, in the case of a whole life insurance policy, it would be usual for the death benefit to be
secured by regular premiums, and it would be common for premium payments to cease at a certain age,
perhaps at age 65, when the PH is assume to retire, or at age 80, when the PH’s real income may be
diminishing.
We can classify the types of premiums in the following way:
According to whether expenses are considered:

• Calculation of premium without IC’s expenses ⇒ net premium (risk and/or mathematical
premium).

• Calculation of premium with explicit IC’s expenses ⇒ gross premium (or office premium).

According to the type (frequency) of premiums, we have

• single payment by PH ⇒ single premium.

• regular series of payments (annually, monthly, weekly, daily, etc.) ⇒ multiple premiums.

Downloaded by Kipkoech Stanley61


(stanleykipkoech8@gmail.com)
lOMoARcPSD|3982611

CHAPTER 5. PREMIUM CALCULATION AND POLICY VALUES 62

The present value of future loss random variable


Cash flows for a traditional life insurance contract (or annuity) are:

• insurance/annuity benefit B (outgo),

• associated IC expenses (outgo), and

• premium P (income).

Note that the three items above are generally life contingent. If a single premium is paid however, then
there is no uncertainty in P . We can now define the future loss random variable as future outgo - future
income. What is more useful however is its present value.
Definition: PV of future loss random variable is

PV (future loss random variable) = PV (future outgo - future income)

Depending on whether expenses are included, we classify the PV of future loss random variable as net
or gross and we will denote it accordingly.
Let us assume that for a life insurance policy the cash flow for the company consists only of premium
income and benefit outgo. Since expenses are excluded we use the “net future loss” which is denoted
by LN0 and defined as

LN
0 = PV of future benefit outgo − PV of net premium income

If expenses are also included, we use the the “gross future loss” which is denoted LG
0 = and defined as

LG
0 = PV(benefit/outgo) + PV (expenses/outgo) − PV (gross premium/income)

Example: An insurer issues a whole life insurance contract to a life currently aged 60 with sum
insurerd S payable at the end of the year of death and premiums P payable annually in advance. Find
an expression for the net future loss random variable.

Solution: From chapter 3 we know that the present value random variable for a whole life insurance
benefit S payable at the end of the year of death to a life aged 60 is

Z = Sv K60 +1

Similarly, from chapter 4 the present value random variable for a whole life annuity due with annual
payments P is
Y = P äK60 +1
Therefore, the net future loss random variable is

LN
0 = PV of future benefits − PV of future premiums
= Sv K60 +1 − P äK60 +1

The equivalence principle


The equivalence principle is a method for determining premiums such that at the start of the contract
the expected present value of the future loss random variable is zero. This means

E[L0 ] = 0

Downloaded by Kipkoech Stanley (stanleykipkoech8@gmail.com)


lOMoARcPSD|3982611

CHAPTER 5. PREMIUM CALCULATION AND POLICY VALUES 63

Therefore, the equivalence principle calculates the premium by equating the expected outgo and income
of the insurance company. When we exclude expenses the equivalence principle takes the form:

E[LN
0 ] = 0 =⇒ E[PV of future benefits − PV of future premiums] = 0

and therefore
EPV of benefits = EPV of premiums (5.1)

When expenses are taken into account the equivalence principle takes the form:

E[LG
0] = 0 =⇒ E[PV(benefit/outgo) + PV (expenses/outgo) − PV (gross premium/income)] = 0

and therefore

EPV of benefits + PV (expenses/outgo) = EPV of premiums (5.2)

Examples of Net Premiums


We now provide a general formula for the annual net premium for whole life and endowment insurance
contracts and also show how the variance of the future loss random variable may be calculated. The
net premium and sum insured are always denoted by P and S respectively.

Whole life insurance


Consider a policy issued to a life currently aged x with sum insured S payable at the end of the year
of death and annual premiums P payable in advance. Using results from chapters 3 and 4, the present
value of future benefits and premiums is

Z = Sv Kx +1 Y = P äKx +1

Therefore, the net future loss random variable is


Kx +1
LN
0 = Sv − P äKx +1

Using the equivalence principle


 Kx +1 
E[LN
 
0 ] = SE v − P E äKx +1 = SAx − P äx = 0

SAx
=⇒ P = (5.3)
äx

Downloaded by Kipkoech Stanley (stanleykipkoech8@gmail.com)


lOMoARcPSD|3982611

CHAPTER 5. PREMIUM CALCULATION AND POLICY VALUES 64

For the variance of the net future loss random variable


1 − v Kx +1
  
Kx +1 Kx +1
Var[LN
 
0 ] = Var Sv − P äKx +1 = Var Sv −P
d
  
P Kx +1 P
= Var S + v −
d d
 2
P
Var v Kx +1
 
= S+
d
 2
P 2
Ax − (Ax )2

= S+
d

Substituting for equation (5.2) gives


 2  2
SAx 2 2 2 däx + Ax 2
Var[LN Ax − (Ax )2
 
0 ] = S+ Ax − (Ax ) =S
däx däx

S 2 (2 Ax − (Ax )2 )
= (using equation (4.7))
(däx )2

S 2 (2 Ax − (Ax )2 )
=⇒ Var[LN
0 ] = (5.4)
(1 − Ax )2

Endowment insurance
Consider an n-year endowment insurance policy issued to a life aged x with sum insured S payable at
the end of the year of death and premiums P payable annually in advance throughout the term of the
contract. The future loss random variable is given by
min(Kx +1,n)
LN
0 = Sv − P ämin(Kx +1,n)

so by the equivalence principle


 min(Kx +1,n)  h i
E[LN
0 ] = SE v − P E ämin(Kx +1,n) = SAx:n − P äx:n = 0

SAx:n
=⇒ P = (5.5)
äx:n

The variance of the net future loss random variable is found in a similar way as in the case of the whole

Downloaded by Kipkoech Stanley (stanleykipkoech8@gmail.com)


lOMoARcPSD|3982611

CHAPTER 5. PREMIUM CALCULATION AND POLICY VALUES 65

life insurance.
1 − v min(Kx +1,n)
h i   
min(Kx +1,n) min(Kx +1,n)
Var[LN
0 ] = Var Sv − P ämin(Kx +1,n) = Var Sv −P
d
 2  2
P  min(Kx +1,n)  P 2
Ax:n − (Ax:n )2

= S+ Var v = S+
d d

 2
SAx:n 2
Ax:n − (Ax:n )2

= S+
däx:n

 2
2 däx:n + Ax:n 2
Ax:n − (Ax:n )2

=S
däx:n

S 2 (2 Ax:n − (Ax:n )2 )
=
(däx:n )2

S 2 (2 Ax:n − (Ax:n )2 )
=⇒ Var[LN
0 ] = (5.6)
(1 − Ax:n )2

We now give the net future loss random variable and premium formula for a term and pure endowment
insurance contract, they can be derived using a similar approach as for a whole life and endowment
contract.

Term insurance:
1
Sv Kx +1 − P äKx +1

if Kx ≤ n − 1 SAx:n
LN
0 = =⇒ P =
−P än if Kx ≥ n äx:n

Pure endowment insurance:



N −P äKx +1 if Kx ≤ n − 1 S n Ex
L0 = =⇒ P =
Sv n − P än if Kx ≥ n äx:n

5.2 The Equivalence principle (continued)


Much of the theory for this chapter has already been covered in the previous lecture so we now briefly
show how a formula for the premium may be derived in the case of a continuous time insurance contract
before providing a number of worked examples. The final lecture consists only of examples.

Whole life insurance (continuous)


Consider a fully continuous insurance contract with sum insured S payable immediately on death and
premiums payable continuously at a rate P during the life of the individual. The net future loss random
variable is
LN0 = Sv
Tx
− P āTx

and by the equivalence principle the premium is given by

E[LN
 Tx   
0 ] = SE v − P E āTx = S Āx − P āx = 0

Downloaded by Kipkoech Stanley (stanleykipkoech8@gmail.com)


lOMoARcPSD|3982611

CHAPTER 5. PREMIUM CALCULATION AND POLICY VALUES 66

S Āx
=⇒ P = (5.7)
āx

The variance of the net future loss random variable is then defined as
1 − v Tx
  
N
 Tx  Tx
Var[L0 ] = Var Sv − P āTx = Var Sv − P
δ
 2
P
Var v Tx
 
= S+
δ

 2
S Āx 2
Āx − (Āx )2

= S+
δāx

 2
2 δāx + Āx 2
Āx − (Āx )2

=S
δāx

S2 2
Āx − (Āx )2

=
(δāx )2

S2 2
Āx − (Āx )2

=⇒ Var[LN
0 ] = 2 (5.8)
1 − Āx
For other fully continuous insurance policies we have

Endowment:

min(Tx ,n) S Āx:n S 2 (2 Āx:n − (Āx:n )2 )


LN
0 = Sv − P āmin(Tx ,n) =⇒ P = , Var[Ln0 ] =
āx:n (1 − Āx:n )2

Term insurance:
1
Sv Tx − P āTx

if Tx ≤ n S Āx:n
LN
0 = =⇒ P =
−P ān if Tx > n āx:n

Example: A life aged 45 purchases a 20-year endowment insurance where a benefit of £10,000 is paid
at the end of the year of death should death occur within 20 years and £20,000 at the end of 20 years
if the insured is still alive. Annual level premiums are paid at the beginning of each year for the first
10 years only and mortality follows the Illustrative Life Table at i=6%. Calculate the annual premium
using the equivalence principle.

Solution: We first need to calculate the present value of the benefits and premiums. Recall that an
endowment insurance is the sum of a pure endowment and term insurance, in this case £10,000 is the
benefit for the term insurance and £20,000 is the benefit for the pure endowment insurance.

10000v K45 +1

if Kx ≤ 19
=⇒ PV of benefits =
20000v 20 if Kx ≥ 20

and therefore the EPV of the benefit is


1
EPV(benefit) = 10000A45:20 + 2000020 E45

Downloaded by Kipkoech Stanley (stanleykipkoech8@gmail.com)


lOMoARcPSD|3982611

CHAPTER 5. PREMIUM CALCULATION AND POLICY VALUES 67

The present value of the premiums is equal to the present value of a 10-year temporary annuity due,
that is,
PV of premiums = P ämin(K45 +1,10)

where P denotes the annual premium. The EPV of the premiums is then given by

EPV(premiums) = P ä45:10

If we now equate the EPV of benefits and premiums as stated by the equivalence principle, we find that
1
1
10000A45:20 + 2000020 E45
10000A45:20 + 2000020 E45 = P ä45:10 =⇒ P =
ä45:10

However, we must now write our temporary annuity and term insurance in terms of whole life annuities
and insurances since these are the only values provided in the Tables.
l65 1 94, 579.73
20 E45 = v 20 = = 0.3599383
l45 1.0520 99, 033.94
l55 1 97, 846.2
10 E45 = v 10 = = 0.6065504
l45 1.0510 99, 033.94
1
A45:20 = A45 − 20 E45 A65 = 0.15161 − (0.3599383 × 0.35477) = 0.0239147
ä45:10 = ä45 − 10 E45 ä55 = 17.81621 − (0.6065504 × 16.05987) = 8.075089
Substituting these values into our expression for the premium gives

(10000 × 0.0239147) + (20000 × 0.3599383)


P = = 921.0936
8.075089
Therefore, for the policyholder to secure the benefits they must pay an annual premium of £786.00.
Example: Kevin and Kira are both actuaries, they use the same policy to price a fully discrete two-year
endowment insurance of £1000 for a life aged x. You are given

• Kevin calculates non-level premiums of £608 for the first year and £350 for the second year

• Kira calculates a level annual premium of π

• d = 0.05

Calculate π.

Solution: If we consider the EPV of the premiums for each actuary, for Kevin we have

EPV(premiums) = 608 + 350vpx

and for Kira


EPV(premiums) = πäx:2

Since both actuaries calculated the premium on the same policy, by the equivalence principle both of
these expressions should equal the EPV of the benefit, in this case given by

EPV(benefits) = 1000Ax:2

Downloaded by Kipkoech Stanley (stanleykipkoech8@gmail.com)


lOMoARcPSD|3982611

CHAPTER 5. PREMIUM CALCULATION AND POLICY VALUES 68

Therefore, the premium calculated by Kira is given by

1000Ax:2 1000(1 − däx:2 )


π= =
äx:2 äx:2

and so we need to evaluate äx:2 using the information we know from Kevin. Using the equivalence
principle and the expression for the EPV of Kevin’s premiums

1000Ax:2 = 1000(1 − däx:2 ) = 608 + 350vpx


n−1
!
608 350 X
k
=⇒ 1 − d(1 + vpx ) = + vpx using äx:n = v k px
1000 1000 k=0
 
608 350
⇐⇒ 1− −d= + d vpx
1000 1000
   
258 350 350
⇐⇒ 1− = + d (1 + vpx ) adding + d to each side
1000 1000 1000
258
1 − 1000 371
⇐⇒ äx:2 = (1 + vpx ) = 350 =
1000
+ 0.05 200

Substituting this value of äx:2 into our expression for π gives


371

1000 1 − 0.05 200
π= 371
 = 489.0836
200

Hence the annual level premium on the contract is £489.0836. Note that this value seems plausible since
we would expect the level premium to lie between the two non-level premiums calculated by Kevin.
Example: An insurance company issues a 15-year deferred life annuity contract to a life aged 50. You
are given

• level monthly premiums P are paid in advance during the deferred period

• an annuity benefit of £25,000 is paid at the beginning of each year the insured is alive starting
from age 65

• mortality follows the Life Table at the end of the notes with i = 5%

• mortality between integer ages follows the UDD assumption.

(i) Write down an expression for the net future loss random variable
(ii) Calculate the monthly premium P
(iii) If an additional benefit of £10,000 is to be paid immediately on death during the deferred period,
how much will the monthly premium increase by?

Solution: (i) To find the net future loss random variable we need to consider the present value of the
benefits and premiums. The benefit is a 15-year deferred annuity due so the present value is

0 if K50 ≤ 14
PV(benefits) =
25000v 15 äK50 +1−15 if K50 ≥ 15

The premiums are payable monthly for the first 15 years unless death occurs beforehand which is how
we define a 15-year temporary annuity due. Remember also that since P is the monthly premium, the

Downloaded by Kipkoech Stanley (stanleykipkoech8@gmail.com)


lOMoARcPSD|3982611

CHAPTER 5. PREMIUM CALCULATION AND POLICY VALUES 69

amount paid each year is 12P . The present value of the premiums may then be written as
 (12)
 12P äK (12) +1
 if K50 ≤ 14
50
PV(premiums) =
(12)

12P ä15 if K50 ≥ 15

Therefore, the net future loss random variable is


 (12)
 −12P äK (12) +1
 if K50 ≤ 14
50
LN0 = PV(benefits) − PV(premiums) =
(12)

25000v 15 äK50 −14 − 12P ä15 if K50 ≥ 15

(ii) To determine the premium we use the equivalence principle so we need to find the EPV of the
benefits and premiums. Consider the benefits first, the EPV of a 15-year deferred annuity due is
denoted by 15| ä50 but since this does not appear in the Illustrative Life Table, we must write it in terms
of a whole life annuity due. We calculate:
l65 1 94, 579.73
15 E50 = v 15 = = 0.4915149.
l50 1.0515 98, 576.37

15| ä50 = 15 E50 ä65 = 0.4615149 × 13.54979 = 7.637975.


(12)
The EPV of a 15-year temporary annuity due payable monthly is given by ä50:15 which may be written
in terms of whole life annuities due as follows
(12) (12) (12)
ä50:15 = ä50 − 15 E50 ä65

(12) (12)
Since the UDD assumption holds, we may apply equation (4.21) to ä50 and ä65 to give
(12)
ä50 = α(12)ä50 − β(12) = (1.000197 × 17.02453) − 0.466508 = 16.56138
(12)
ä65 = α(12)ä65 − β(12) = (1.000197 × 13.54979) − 0.466508 = 13.08595

The value of the 15-year temporary annuity due is therefore


(12)
ä50:15 = 16.56138 − (0.4915149 × 13.08595) = 10.52201.

If we equate the EPV of the benefits and premiums

(12) 2500015| ä50 25000 × 7.637975


=⇒ 2500015| ä50 = 12P ä50:15 ⇐⇒ P = (12)
= = 1512.301
12ä50:15 12 × 10.52201

Therefore the monthly premium is P = £1512.301.

(iii) If an additional benefit is payable immediately on death during the deferred period, this only has
an effect on the EPV of the benefits and not the premiums. There are also no changes to the benefit
after the deferred period so the total EPV of the benefits will include just a single additional term.

Downloaded by Kipkoech Stanley (stanleykipkoech8@gmail.com)


lOMoARcPSD|3982611

CHAPTER 5. PREMIUM CALCULATION AND POLICY VALUES 70

The present value of the benefits is now given by

10000v T50

if K50 ≤ 14
PV(benefits) = 15
25000v äK50 −14 if K50 ≥ 15

10000v T50
 
0 if K50 ≤ 14 if K50 ≤ 14
= +
25000v 15 äK50 −14 if K50 ≥ 15 0 if K50 ≥ 15

and hence the EPV of the benefits is


1
EPV(benefits) = 2500015| ä50 + 10000Ā50:15
1
We already know the value of 15| ä50 from the second part so we need to evaluate Ā50:15 .

1 i 1 i
Ā50:15 = A50:15 = (A50 − 15 E50 A65 )
δ δ
0.05
= (0.18931 − (0.4915149 × 0.35477))
ln(1.05)
= 0.0262126

If we equate the EPV of the benefits and the premiums and let P ∗ denote the new monthly premium
1 (12)
=⇒ 2500015| ä50 + 10000Ā50:15 = 12P ∗ ä50:15

1

2500015| ä50 + 10000Ā50:15 (25000 × 7.637975) + (10000 × 0.0262126)
⇐⇒ P = (12)
= = 1514.173
12ä50:15 12 × 10.5229

Therefore, the revised monthly premium is P ∗ = £1514.173, an increase of £2 from the premium
without the additional benefit.

Example: For a fully continuous n-year endowment insurance of £1 issued to a life aged x you are
given

• Z is the present value random variable of the benefit for this insurance

• E[Z] = 0.5198

• Var[Z] = 0.1068

• Continuous premium is paid at a rate P per year during the term of the contract.

Calculate Var[LN N
0 ] where L0 is the net future loss random variable.

Solution: For a fully continuous n-year endowment insurance with unit benefit and annual premium
rate P , the net future loss random variable is

min(Tx ,n)
LN
0 = v − P āmin(Tx ,n)

Therefore, the variance is given by


2 2
1 − v min(Tx ,n)
    
N min(Tx ,n) P  min(Tx ,n)  P
Var[L0 ] = Var v −P = + 1 Var v = + 1 Var[Z]
δ δ δ

Downloaded by Kipkoech Stanley (stanleykipkoech8@gmail.com)


lOMoARcPSD|3982611

CHAPTER 5. PREMIUM CALCULATION AND POLICY VALUES 71

Var[Z] is given in the question so we need to find the value of P and δ, however, we are instead going
to find an expression for Pδ . Since the premium is calculated using the equivalence principle

P
=⇒ P āx:n = Āx:n ⇐⇒ (1 − Āx:n ) = Āx:n
δ

P Āx:n E[Z] 0.5198


⇐⇒ = = = = 1.0825
δ 1 − Āx:n 1 − E[Z] 1 − 0.5198

Hence,
2
Var[LN
0 ] = (1.0825 + 1) × 0.1068 = 0.4632

5.3 Policy Values and Reserves


Until now, we have seen how to calculate a “fair” premium for an insurance policy based on the
Equivalence Principle. This premium, once calculated at time t = 0 cannot be changed (unless otherwise
specified in the contract). But how does the insurance company use these premiums? The most obvious
answer is to pay potential claims. But how much of the premiums (or the company’s liquid assets more
generally) should be set aside to cover for insurance claims? To answer this question we need to
introduce the notion of ”Policy Value” and/or ”Reserve”. The two terms can be equal under some
conditions but this is not always true. Here we will focus on ”Policy Values” and more details will be
discussed in Actuarial Mathematics 2.

Every life insurance company needs a balance sheet showing the value of its assets, A, and its liabilities
L. If A ≥ L ⇒ company is solvent whereas if A < L ⇒ company is insolvent.

The insurer has to quantify its assets and liabilities. This is not always straightforward.

• (i) assets: are easy to quantify (market value). Investments (such as bonds, equities and property).
They have been purchased with premiums and they will be held in a fund from which benefits will
be paid out. (An insurer will also be concerned about liquidity risk, i.e. how easily and non-costly
investments are turned to cash.)

• (ii) liabilities: are difficult to quantify. Promises made to pay benefits in the future, against which
can be set the policyholder’s promises to pay premiums in the future.

Question: How do we quantify or value the liabilities?


Answer: Valuation basis; make estimates for future investment rates i%, mortality and possible
expenses, and calculate the assets required to meet the expected future payments to policyholders
⇒ policy value.
Definition: Reserve = the portfolio of assets held by an insurance company in order to ensure that it
can meet its future liabilities. The reserve must be, at least, equal to the total of policy values.
The insurance needs to hold a reserve in respect of a life insurance policy because the premium income
does not coincide with the benefits and expenses outgo.
Example 1: whole life policy issued to (x) with B = £1, payable at the e.o.y of death, (ultimate
mortality is assumed), and level annual premiums, if we ignore expenses.
Solution:
Ax
P = äx
.
E (PV benefits) = E (PV premiums) at time of purchase.
If we make use of the equivalence principle, we have:

Downloaded by Kipkoech Stanley (stanleykipkoech8@gmail.com)


lOMoARcPSD|3982611

CHAPTER 5. PREMIUM CALCULATION AND POLICY VALUES 72

E v Kx +1 − Px äKx +1 = 0.


Kx +1
Remember that LN
0 = v − Px äKx +1 = future loss at the initial time, t = 0.
At any time t > 0 (time in the future), and if the policy is still in place, let us introduce and define

LN
t ,

the future loss at time t.


Note that this is the difference between the present values, at time t, of future outgo and of future
income.

E LNt is called the prospective policy value (or reserve) of the contract at time t, and is denoted t V .
defKx+t +1
LN
t = v − Px äKx+t +1 ⇒

E LN
t = t V = E[LN t ] = Ax+t − Px äx+t .
.

Some notes on policy values


• (i) convention: t V at time t, policy value is calculated just before premium due dates (integer
policy duration). In practice, a life office will calculate the policy values of its “in force” portfolio
on a fixed calendar date, and only by coincidence will this time be an integer policy duration, for
any randomly chosen policy.
• (ii) policy values (and reserves):
def
– policy value denoted by t V = E[LN
t ].

It is calculated by actuary on the basis of some assumptions (i%, µx , etc.)


– reserve (or provision) is defined as the quantity of “actual” assets whose value, at least equals,
the policy value.

• (iii) applications of policy values and reserves:

– 1) if policy alteration,
– 2) demonstrating solvency,
– 3) share holder ⇒ determine dividends to be paid,
– 4) if with-profit policies ⇒ determine bonus levels,
– 5) if policy terms allow the payment of a surrender benefit, needed to pay surrender values
to PH who surrenders (⇒ pay surrender values to PH).

• (iv) valuation basis: interest, mortality, expenses, etc.


• (v) premium calculation: calculation of the premium is a special case of the calculation of policy
values.
If equivalence principle ⇒ E[LN
0 ] = 0. Expected future loss at outset is zero.

Note that by construction E[LN


0 ] = 0V .

Back to Example 1:

E LN N
t=0 = 0 V = E[L0 ] = Ax − P äx .
Ax
⇒P = äx
and 0 V = 0.


Downloaded by Kipkoech Stanley (stanleykipkoech8@gmail.com)


lOMoARcPSD|3982611

CHAPTER 5. PREMIUM CALCULATION AND POLICY VALUES 73

• (vi) boundary conditions for policy values at outset and at expiry of the policy:

– if the basis is the same to compute premium and policy value


⇒ 0 V = 0.
– if a policy does not pay a benefit at maturity
⇒ n V = 0 (n-year term insurance policy).
– for a policy paying out a maturity benefit B after the expiry at time n (maturity benefit at
the end of term n)
⇒ n V = B (n-year endowment).

The prospective policy value (or prospective reserve) for a life insurance contract, which is in force (not
yet expired through claim or reached the end of the term) is defined, for a given basis.
Definition: expected PV (future outgo) - expected PV (future income).
We observe that it is the minimum funds the IC needs to hold at any time during the term of the
contract.
E (PV future outgo)t - E (PV future income)t , where t > 0, t = 0 at the start of the contract.

Net premium policy value (or reserves)


Example: let us consider a n-year endowment insurance policy (policy value at time t > 0), and t Vx:n
is the net premium policy value at time t, with benefit B = £1, payable at the end of the year of death
or at maturity, and with level annual premiums payable during the term of the policy.
Solution: at time t, if policy is in force, we can write

def
LNt = B v X − P äX ,
X = min(Kx+t + 1, n − t) ,
E[LN
t ] = t Vx:n = B Ax+t:n−t − P äx+t:n−t .

We know that:

E[LN t=0 ] = t=0 Vx:n = B Ax:n − P äx:n = 0 ,


Ax:n = 1 − d äx:n ,
Ax+t:n−t = 1 − d äx+t:n−t ,
Ax:n
P = B Px:n = B .
äx:n
Ax:n
⇒ t Vx:n = B Ax+t:n−t − P äx+t:n−t = B Ax+t:n−t − B ä
äx:n x+t:n−t
Ax:n
= B (1 − d äx+t:n−t ) − B ä
äx:n x+t:n−t
 
(1 − d äx:n ) äx+t:n−t
= B (1 − d äx+t:n−t ) − B äx+t:n−t = B 1 − .
äx:n äx:n

If we are interested in computing the variance, we need to write

Downloaded by Kipkoech Stanley (stanleykipkoech8@gmail.com)


lOMoARcPSD|3982611

CHAPTER 5. PREMIUM CALCULATION AND POLICY VALUES 74

 
Px:n Px:n B Px:n
LN X X
vX −

t = B v −B 1−v =B 1+ ,
d d d

where we have made use of the fact


1 − vX
äX = .
d

We conclude that the variance is given by

 2
2 Px:n
V [LN
t ] = B 1+ [2 Ax+t:n−t − (Ax+t:n−t )2 ] .
d

Gross Prospective Reserves


In practice, Net Premium is never the actual premium charged. It is however used for net reserving
purposes. The actual premium is the gross premium that takes into account expenses (and more that
you will discuss in Actuarial Mathematics 2).
The Gross Prospective Reserves at duration t of the policy is calculated as:

G
tV = EPV(future benefits)t + EPV (future expenses/outgo)t − EPV(future premiums)t

Example: Consider a 20−year endowment issued at a life aged exactly 40 years old. The benefit is
£100, 000 payable at the end of year of death (or maturity).The policy incurs initial expenses of £250,
renewal expenses equal to 10% of the annual premium and £100 termination (at maturity or at benefit
payment) expenses. Calculate the gross prospective reserve at t = 5.
We first need to calculate the gross premium using the equivalence principle.

EPV(benefit) = 100, 000A40:20


1
= 100, 000(A40:20 + 20 E45 )
l60 l60
= 100, 000(A40 − v 20 A60 + v 20 ) = 38, 126.47
l40 l40

EPV(expenses) = 250 + P 0.1(ä40:20 − 1) + 100A40:20


l60
= 250 + 0.1P (ä40 − v 20 ä60 − 1) + 100A40:20
l40
= 288.1265 + 1.99348P

EPV(premium) = P ä40:20 = 12.99348P


Then, from the equivalence principle we get that P = 3, 257.094.

Downloaded by Kipkoech Stanley (stanleykipkoech8@gmail.com)


lOMoARcPSD|3982611

CHAPTER 5. PREMIUM CALCULATION AND POLICY VALUES 75

Now we are ready to calculate the reserve at time t = 5.


G
5V = EPV(benefits)t=5 + EPV (expenses/outgo)t=5 − EPV(premiums)t=5

EPV(benefit)t = 5 = 100, 000A45:15


1
= 100, 000(A40:20 + 20 E45 )
l60 l60
= 100, 000(A45 − v 15 A60 + v 15 ) = 48, 472.49
l45 l45

EPV(expenses)t=5 = P 0.1(ä45:15 − 1) + 100A45:15


l60
= 0.1P (ä45 − v 15 ä60 − 1) + 100A45:15
l45
= 3, 247.206

EPV(premium)t=5 = P ä45:15 = 35, 244.43


Therefore, the gross reserve will be 5 V G = 16, 457.27.

Downloaded by Kipkoech Stanley (stanleykipkoech8@gmail.com)


lOMoARcPSD|3982611

Appendix

Actuarial Tables for University of Leeds Examinations 2016/17


i=5%

x l[x] l[x]+1 lx+2 A[x] A[x]+1 Ax+2 (IA)x+2 x+2


28 99, 781.36 99, 756.17 99, 727.29 0.07032 0.07357 0.07698 3.652248809 30
29 99, 751.69 99, 725.70 99, 695.83 0.07356 0.07697 0.08054 3.755213856 31
30 99, 721.06 99, 694.18 99, 663.20 0.07696 0.08053 0.08427 3.859668736 32
31 99, 689.36 99, 661.48 99, 629.26 0.08051 0.08425 0.08817 3.965520005 33
32 99, 656.47 99, 627.47 99, 593.83 0.08424 0.08816 0.09226 4.072663183 34
33 99, 622.23 99, 591.96 99, 556.75 0.08814 0.09224 0.09653 4.180982148 35
34 99, 586.47 99, 554.78 99, 517.80 0.09223 0.09652 0.10101 4.290348521 36
35 99, 549.01 99, 515.73 99, 476.75 0.09650 0.10099 0.10569 4.400621053 37
36 99, 509.64 99, 474.56 99, 433.34 0.10097 0.10567 0.11059 4.511645019 38
37 99, 468.12 99, 431.02 99, 387.29 0.10565 0.11057 0.11571 4.623251623 39
38 99, 424.18 99, 384.82 99, 338.26 0.11055 0.11569 0.12106 4.735257429 40
39 99, 377.52 99, 335.62 99, 285.88 0.11567 0.12104 0.12665 4.847463812 41
40 99, 327.82 99, 283.06 99, 229.76 0.12101 0.12663 0.13249 4.959656446 42
41 99, 274.69 99, 226.72 99, 169.41 0.12660 0.13247 0.13859 5.071604847 43
42 99, 217.72 99, 166.14 99, 104.33 0.13244 0.13857 0.14496 5.183061963 44
43 99, 156.42 99, 100.80 99, 033.94 0.13854 0.14493 0.15161 5.293763836 45
44 99, 090.27 99, 030.10 98, 957.57 0.14491 0.15158 0.15854 5.403429353 46
45 99, 018.67 98, 953.40 98, 874.50 0.15155 0.15851 0.16577 5.511760086 47
46 98, 940.96 98, 869.96 98, 783.91 0.15847 0.16573 0.17330 5.618440252 48
47 98, 856.38 98, 778.94 98, 684.88 0.16569 0.17326 0.18114 5.723136801 49
48 98, 764.09 98, 679.44 98, 576.37 0.17322 0.18110 0.18931 5.825499653 50
49 98, 663.15 98, 570.40 98, 457.24 0.18106 0.18926 0.19780 5.925162103 51
50 98, 552.51 98, 450.67 98, 326.19 0.18921 0.19775 0.20664 6.02174142 52
51 98, 430.98 98, 318.95 98, 181.77 0.19770 0.20658 0.21582 6.114839644 53
52 98, 297.24 98, 173.79 98, 022.38 0.20653 0.21576 0.22535 6.20404463 54
53 98, 149.81 98, 013.56 97, 846.20 0.21570 0.22529 0.23524 6.28893133 55
54 97, 987.03 97, 836.44 97, 651.21 0.22522 0.23517 0.24550 6.369063361 56
55 97, 807.07 97, 640.40 97, 435.17 0.23510 0.24542 0.25613 6.443994859 57
56 97, 607.84 97, 423.18 97, 195.56 0.24534 0.25605 0.26714 6.513272655 58
57 97, 387.05 97, 182.25 96, 929.59 0.25596 0.26704 0.27852 6.576438778 59
58 97, 142.13 96, 914.80 96, 634.14 0.26695 0.27842 0.29028 6.633033303 60
59 96, 870.22 96, 617.70 96, 305.75 0.27831 0.29017 0.30243 6.682597561 61
60 96, 568.13 96, 287.48 95, 940.60 0.29005 0.30230 0.31495 6.72467771 62
61 96, 232.34 95, 920.27 95, 534.43 0.30217 0.31481 0.32785 6.758828664 63
62 95, 858.91 95, 511.80 95, 082.53 0.31467 0.32770 0.34113 6.784618387 64
63 95, 443.51 95, 057.36 94, 579.73 0.32755 0.34097 0.35477 6.801632525 65
64 94, 981.34 94, 551.72 94, 020.33 0.34080 0.35459 0.36878 6.809479354 66
65 94, 467.11 93, 989.16 93, 398.05 0.35441 0.36858 0.38313 6.807795016 67
66 93, 895.00 93, 363.38 92, 706.06 0.36838 0.38292 0.39783 6.796248988 68
67 93, 258.63 92, 667.50 91, 936.88 0.38270 0.39760 0.41285 6.774549735 69

Downloaded by Kipkoech Stanley76


(stanleykipkoech8@gmail.com)
lOMoARcPSD|3982611

CHAPTER 5. PREMIUM CALCULATION AND POLICY VALUES 77

x l[x] l[x]+1 lx+2 A[x] A[x]+1 Ax+2 (IA)x+2 x+2


68 92, 551.02 91, 894.03 91, 082.43 0.39736 0.41260 0.42818 6.742450463 70
69 91, 764.58 91, 034.84 90, 133.96 0.41234 0.42790 0.44379 6.699754898 71
70 90, 891.07 90, 081.15 89, 082.09 0.42762 0.44349 0.45968 6.646322984 72
71 89, 921.62 89, 023.56 87, 916.84 0.44319 0.45935 0.47580 6.582076391 73
72 88, 846.72 87, 852.03 86, 627.64 0.45902 0.47545 0.49215 6.507003704 74
73 87, 656.25 86, 555.99 85, 203.46 0.47509 0.49177 0.50868 6.421165157 75
74 86, 339.55 85, 124.37 83, 632.89 0.49138 0.50826 0.52536 6.324696777 76
75 84, 885.49 83, 545.75 81, 904.34 0.50785 0.52492 0.54217 6.21781377 77
76 83, 282.61 81, 808.54 80, 006.23 0.52447 0.54169 0.55906 6.100813016 78
77 81, 519.30 79, 901.17 77, 927.35 0.54121 0.55854 0.57599 5.974074522 79
78 79, 584.04 77, 812.44 75, 657.16 0.55802 0.57544 0.59293 5.838061685 80
79 77, 465.70 75, 531.88 73, 186.31 0.57488 0.59234 0.60984 5.693320261 81
80 75, 153.97 73, 050.22 70, 507.19 0.59175 0.60920 0.62666 5.540475924 82
81 72, 639.81 70, 359.94 67, 614.60 0.60857 0.62598 0.64336 5.380230363 83
82 69, 916.06 67, 455.99 64, 506.50 0.62532 0.64264 0.65990 5.213355855 84
83 66, 978.12 64, 336.53 61, 184.88 0.64193 0.65913 0.67622 5.040688357 85
84 63, 824.72 61, 003.79 57, 656.68 0.65838 0.67540 0.69229 4.86311913 86
85 60, 458.83 57, 464.98 53, 934.73 0.67462 0.69142 0.70806 4.681585034 87
86 56, 888.53 53, 733.29 50, 038.65 0.69060 0.70714 0.72349 4.49705762 88
87 53, 128.02 49, 828.70 45, 995.64 0.70629 0.72252 0.73853 4.310531237 89
88 49, 198.45 45, 778.84 41, 841.05 0.72163 0.73752 0.75317 4.123010406 90
89 45, 128.71 41, 619.51 37, 618.56 0.73660 0.75211 0.76735 3.935496741 91
90 40, 955.90 37, 394.80 33, 379.88 0.75115 0.76624 0.78104 3.748975755 92
91 36, 725.43 33, 156.84 29, 183.78 0.76526 0.77989 0.79423 3.564403889 93
92 32, 490.61 28, 964.73 25, 094.33 0.77889 0.79303 0.80688 3.382696112 94
93 28, 311.50 24, 882.75 21, 178.30 0.79201 0.80564 0.81897 3.204714446 95
94 24, 252.97 20, 977.73 17, 501.76 0.80460 0.81769 0.83049 3.031257728 96
95 20, 381.98 17, 315.59 14, 125.89 0.81665 0.82918 0.84143 2.863052897 97
96 16, 763.92 13, 957.13 11, 102.53 0.82814 0.84009 0.85177 2.70074804 98
97 13, 458.37 10, 953.56 8, 469.73 0.83905 0.85041 0.86153 2.54490735 99
98 10, 514.62 8, 342.12 6, 248.17 0.84938 0.86014 0.87068 2.396008116 100
99 7, 967.35 6, 142.47 4, 438.80 0.85913 0.86928 0.87925 2.254439739 101
100 5, 833.20 4, 354.49 3, 022.58 0.86830 0.87784 0.88724 2.120504722 102
101 4, 108.86 2, 958.13 1, 962.49 0.87690 0.88582 0.89465 1.994421504 103
102 2, 771.25 1, 915.52 1, 207.79 0.88492 0.89323 0.90150 1.87632892 104
103 1, 780.05 1, 175.35 699.91 0.89239 0.90010 0.90781 1.766292023 105
104 1, 082.34 678.82 379.08 0.89931 0.90643 0.91360 1.664308939 106
105 618.75 366.26 190.28 0.90571 0.91224 0.91888 1.570318406 107
106 330.04 183.07 87.69 0.91159 0.91756 0.92367 1.484207571 108
107 162.85 83.97 36.71 0.91698 0.92240 0.92800 1.405819641 109
108 73.61 34.96 13.80 0.92189 0.92678 0.93188 1.334960925 110
109 30.16 13.06 4.59 0.92634 0.93072 0.93534 1.271406814 111
110 11.06 4.32 1.33 0.93036 0.93426 0.93840 1.214906225 112
111 3.58 1.24 0.33 0.93396 0.93739 0.94107 1.165184082 113
112 1.01 0.31 0.07 0.93716 0.94016 0.94339 1.121941457 114
113 0.24 0.06 0.01 0.93998 0.94257 0.94537 1.08485318 115
114 0.05 0.01 0.00 0.94244 0.94464 0.94703 1.053563039 116
115 0.01 0.00 0.00 0.94455 0.94641 0.94841 1.0276772 117

Downloaded by Kipkoech Stanley (stanleykipkoech8@gmail.com)


lOMoARcPSD|3982611

CHAPTER 5. PREMIUM CALCULATION AND POLICY VALUES 78

x ä[x] ä[x]+1 äx+2 (Iä)[x] (Iä)[x]+1 (Iä)x+2 x + 2


30 19.38449 19.30892 19.23034 330.37389 326.62691 322.78419 32
31 19.30979 19.23066 19.14838 326.64260 322.78973 318.84013 33
32 19.23156 19.14871 19.06258 322.80588 318.84584 314.78826 34
33 19.14965 19.06292 18.97277 318.86251 314.79415 310.62762 35
34 19.06390 18.97313 18.87880 314.81141 310.63373 306.35745 36
35 18.97415 18.87917 18.78049 310.65164 306.36380 301.97715 37
36 18.88024 18.78088 18.67766 306.38244 301.98377 297.48630 38
37 18.78201 18.67807 18.57014 302.00322 297.49322 292.88473 39
38 18.67927 18.57058 18.45776 297.51358 292.89198 288.17248 40
39 18.57184 18.45822 18.34031 292.91336 288.18010 283.34985 41
40 18.45956 18.34081 18.21763 288.20260 283.35788 278.41741 42
41 18.34224 18.21815 18.08951 283.38162 278.42589 273.37601 43
42 18.21969 18.09007 17.95577 278.45100 273.38499 268.22685 44
43 18.09172 17.95637 17.81621 273.41163 268.23638 262.97143 45
44 17.95814 17.81686 17.67065 268.26469 262.98157 257.61163 46
45 17.81876 17.67135 17.51889 263.01173 257.62244 252.14969 47
46 17.67340 17.51965 17.36074 257.65462 252.16123 246.58827 48
47 17.52187 17.36156 17.19602 252.19564 246.60062 240.93045 49
48 17.36397 17.19691 17.02453 246.63745 240.94367 235.17974 50
49 17.19952 17.02551 16.84612 240.98317 235.19392 229.34013 51
50 17.02835 16.84718 16.66060 235.23632 229.35536 223.41609 52
51 16.85028 16.66175 16.46782 229.40091 223.43245 217.41258 53
52 16.66514 16.46908 16.26762 223.48144 217.43017 211.33509 54
53 16.47277 16.26899 16.05987 217.48289 211.35402 205.18964 55
54 16.27303 16.06137 15.84443 211.41077 205.21001 198.98279 56
55 16.06579 15.84608 15.62122 205.27112 199.00472 192.72165 57
56 15.85091 15.62302 15.39012 199.07053 192.74525 186.41388 58
57 15.62831 15.39210 15.15109 192.81612 186.43928 180.06769 59
58 15.39789 15.15325 14.90407 186.51557 180.09501 173.69186 60
59 15.15960 14.90644 14.64906 180.17711 173.72124 167.29568 61
60 14.91340 14.65165 14.38606 173.80954 167.32725 160.88898 62
61 14.65927 14.38890 14.11512 167.42215 160.92287 154.48209 63
62 14.39724 14.11822 13.83632 161.02479 154.51844 148.08579 64
63 14.12736 13.83972 13.54979 154.62778 148.12474 141.71131 65
64 13.84972 13.55351 13.25568 148.24192 141.75300 135.37026 66
65 13.56444 13.25975 12.95420 141.87843 135.41483 129.07459 67
66 13.27169 12.95864 12.64561 135.54890 129.12216 122.83651 68
67 12.97168 12.65045 12.33019 129.26527 122.88721 116.66844 69
68 12.66467 12.33547 12.00830 123.03972 116.72238 110.58291 70
69 12.35097 12.01406 11.68035 116.88464 110.64020 104.59250 71
70 12.03093 11.68661 11.34678 110.81252 104.65322 98.70970 72

Downloaded by Kipkoech Stanley (stanleykipkoech8@gmail.com)


lOMoARcPSD|3982611

CHAPTER 5. PREMIUM CALCULATION AND POLICY VALUES 79

x ä[x] ä[x]+1 äx+2 (Iä)[x] (Iä)[x]+1 (Iä)x+2 x + 2


71 11.70495 11.35359 11.00812 104.83588 98.77394 92.94687 73
72 11.37350 11.01550 10.66491 98.96716 93.01470 87.31608 74
73 11.03709 10.67291 10.31778 93.21865 87.38753 81.82901 75
74 10.69629 10.32644 9.96740 87.60234 81.90411 76.49687 76
75 10.35171 9.97676 9.61449 82.12983 76.57561 71.33021 77
76 10.00402 9.62458 9.25981 76.81221 71.41257 66.33889 78
77 9.65395 9.27067 8.90416 71.65994 66.42480 61.53188 79
78 9.30225 8.91584 8.54841 66.68276 61.62127 56.91722 80
79 8.94973 8.56093 8.19341 61.88954 57.00997 52.50189 81
80 8.59722 8.20681 7.84008 57.28817 52.59784 48.29171 82
81 8.24560 7.85440 7.48934 52.88550 48.39069 44.29128 83
82 7.89575 7.50460 7.14211 48.68723 44.39307 40.50392 84
83 7.54859 7.15834 6.79934 44.69784 40.60828 36.93161 85
84 7.20502 6.81655 6.46193 40.92050 37.03827 33.57497 86
85 6.86596 6.48015 6.13079 37.35707 33.68364 30.43327 87
86 6.53231 6.15002 5.80679 34.00808 30.54362 27.50445 88
87 6.20493 5.82705 5.49077 30.87267 27.61612 24.78512 89
88 5.88469 5.51204 5.18352 27.94869 24.89776 22.27067 90
89 5.57237 5.20579 4.88575 25.23268 22.38390 19.95536 91
90 5.26873 4.90900 4.59814 22.71998 20.06878 17.83235 92
91 4.97447 4.62234 4.32126 20.40476 17.94558 15.89391 93
92 4.69020 4.34637 4.05562 18.28021 16.00655 14.13150 94
93 4.41647 4.08161 3.80166 16.33856 14.24315 12.53592 95
94 4.15375 3.82845 3.55971 14.57128 12.64622 11.09749 96
95 3.90242 3.58723 3.33001 12.96923 11.20605 9.80615 97
96 3.66279 3.35819 3.11273 11.52274 9.91263 8.65166 98
97 3.43504 3.14146 2.90794 10.22185 8.75572 7.62373 99
98 3.21930 2.93713 2.71563 9.05637 7.72505 6.71212 100
99 3.01561 2.74516 2.53572 8.01611 6.81042 5.90684 101
100 2.82392 2.56545 2.36804 7.09095 6.00184 5.19818 102
101 2.64410 2.39784 2.21237 6.27099 5.28965 4.57686 103
102 2.47597 2.24210 2.06843 5.54664 4.66457 4.03408 104
103 2.31927 2.09792 1.93589 4.90874 4.11782 3.56158 105
104 2.17370 1.96497 1.81439 4.34862 3.64118 3.15168 106
105 2.03893 1.84289 1.70352 3.85814 3.22697 2.79731 107
106 1.91458 1.73125 1.60288 3.42975 2.86816 2.49204 108
107 1.80025 1.62965 1.51201 3.05652 2.55830 2.23002 109
108 1.69554 1.53764 1.43049 2.73213 2.29158 2.00601 110
109 1.60004 1.45478 1.35785 2.45087 2.06278 1.81535 111
110 1.51334 1.38063 1.29366 2.20762 1.86724 1.65391 112
111 1.43504 1.31475 1.23747 1.99785 1.70086 1.51803 113
112 1.36476 1.25672 1.18882 1.81756 1.56002 1.40453 114
113 1.30212 1.20611 1.14726 1.66322 1.44157 1.31061 115
114 1.24676 1.16250 1.11232 1.53178 1.34275 1.23380 116
115 1.19833 1.12546 1.08348 1.42056 1.26116 1.17190 117
116 1.15647 1.09454 1.06023 1.32725 1.19468 1.12296 118
117 1.12083 1.06927 1.04199 1.24981 1.14141 1.08515 119
118 1.09102 1.04913 1.02814 1.18642 1.09964 1.05677 120

Downloaded by Kipkoech Stanley (stanleykipkoech8@gmail.com)

Potrebbero piacerti anche